cases & projects

80

Click here to load reader

Upload: hoangdien

Post on 16-Dec-2016

290 views

Category:

Documents


7 download

TRANSCRIPT

Page 1: Cases & Projects

48 Chapter 1 Introduction to Accounting and Business48 Chapter 1 Introduction to Accounting and Business

CP 1-1 Ethics and professional conduct in businessGroup ProjectVince Hunt, president of Sabre Enterprises, applied for a $200,000 loan from First Na-tional Bank. The bank requested financial statements from Sabre Enterprises as a basis for granting the loan. Vince has told his accountant to provide the bank with a balance sheet. Vince has decided to omit the other financial statements because there was a net loss during the past year.

In groups of three or four, discuss the following questions:

1. Is Vince behaving in a professional manner by omitting some of the financial state-ments?

2. a. What types of information about their businesses would owners be willing to pro-vide bankers? What types of information would owners not be willing to provide?

b. What types of information about a business would bankers want before extending a loan?

c. What common interests are shared by bankers and business owners?

CP 1-2 Net incomeOn July 1, 2011, Dr. Heather Dewitt established Life Medical, a medical practice organized as a professional corporation. The following conversation occurred the following January between Dr. Dewitt and a former medical school classmate, Dr. Naomi Kennedy, at an American Medical Association convention in Boston.

Dr. Kennedy: Heather, good to see you again. Why didn’t you call when you were in Chicago? We could have had dinner together.

Dr. Dewitt: Actually, I never made it to Chicago this year. My husband and kids went up to our Vail condo twice, but I got stuck in Fort Lauderdale. I opened a new consulting practice this July and haven’t had any time for myself since.

Dr. Kennedy: I heard about it . . . Life . . . something . . . right?

Dr. Dewitt: Yes, Life Medical. My husband chose the name.

Dr. Kennedy: I’ve thought about doing something like that. Are you making any money? I mean, is it worth your time?

Dr. Dewitt: You wouldn’t believe it. I started by opening a bank account with $40,000, and my December bank state-ment has a balance of $90,000. Not bad for six months—all pure profi t.

Dr. Kennedy: Maybe I’ll try it in Chicago! Let’s have breakfast together tomorrow and you can fi ll me in on the details.

Comment on Dr. Dewitt’s statement that the difference between the opening bank balance ($40,000) and the December statement balance ($90,000) is pure profit.

CP 1-3 Transactions and financial statementsJan Martinelli, a junior in college, has been seeking ways to earn extra spending money. As an active sports enthusiast, Jan plays tennis regularly at the Naples Tennis Club, where her family has a membership. The president of the club recently approached Jan with the proposal that she manage the club’s tennis courts. Jan’s primary duty would be to supervise the operation of the club’s four indoor and 10 outdoor courts, including court reservations.

In return for her services, the club would pay Jan $300 per week, plus Jan could keep whatever she earned from lessons and the fees from the use of the ball machine. The club and Jan agreed to a one-month trial, after which both would consider an arrangement for the remaining two years of Jan’s college career. On this basis, Jan organized Topspin. Dur-ing April 2012, Jan managed the tennis courts and entered into the following transactions:

a. Opened a business account by depositing $1,000.

b. Paid $300 for tennis supplies (practice tennis balls, etc.).

c. Paid $200 for the rental of video equipment to be used in offering lessons during April.

d. Arranged for the rental of two ball machines during April for $250. Paid $100 in ad-vance, with the remaining $150 due May 1.

Cases & Projects

amazzaro
Text Box
Chapter 1
Page 2: Cases & Projects

Chapter 1 Introduction to Accounting and Business 49Chapter 1 Introduction to Accounting and Business 49

e. Received $1,600 for lessons given during April.

f. Received $500 in fees from the use of the ball machines during April.

g. Paid $800 for salaries of part-time employees who answered the telephone and took reservations while Jan was giving lessons.

h. Paid $225 for miscellaneous expenses.

i. Received $1,200 from the club for managing the tennis courts during April.

j. Determined that the cost of supplies on hand at the end of the month totaled $180; therefore, the cost of supplies used was $120.

k. Withdrew $250 for personal use on April 30.

As a friend and accounting student, you have been asked by Jan to aid her in assess-ing the venture.

1. Indicate the effect of each transaction and the balances after each transaction, using the following tabular headings:

Assets = Liabilities + Owner’s Equity

Cash + Supplies =Accounts Payable +

Jan Martinelli,

Capital –

Jan Martinelli, Drawing +

Service Revenue –

Salary Expense –

Rent Expense –

Supplies Expense –

Misc. Expense

2. Prepare an income statement for April.

3. Prepare a statement of owner’s equity for April. The statement of owner’s equity for a proprietorship is similar to the retained earnings statement for a corporation. The balance of the owner’s capital as of the beginning of the period is listed first. Any investments made by the owner during the period are then listed and the net income (net loss) is added (subtracted) to determine a subtotal. From this subtotal, the owner’s withdrawals are subtracted to determine the increase (decrease) in owner’s equity for the period. This increase (decrease) is then added to (subtracted from) the beginning owner’s equity to determine the owner’s equity as of the end of the period.

4. Prepare a balance sheet as of April 30.

5. a. Assume that Jan Martinelli could earn $9 per hour working 30 hours a week as a waitress. Evaluate which of the two alternatives, working as a waitress or operating Topspin, would provide Jan with the most income per month.

b. Discuss any other factors that you believe Jan should consider before discussing a long-term arrangement with the Naples Tennis Club.

CP 1-4 Certification requirements for accountantsBy satisfying certain specific requirements, accountants may become certified as public accountants (CPAs), management accountants (CMAs), or internal auditors (CIAs). Find the certification requirements for one of these accounting groups by accessing the ap-propriate Internet site listed below.

Site Descriptionhttp://www.ais-cpa.com This site lists the address and/or Internet link for each state’s board of accoun-

tancy. Find your state’s requirements.http://www.imanet.org This site lists the requirements for becoming a CMA.http://www.theiia.org This site lists the requirements for becoming a CIA.

CP 1-5 Cash flowsAmazon.com, an Internet retailer, was incorporated and began operation in the mid-90s. On the statement of cash flows, would you expect Amazon.com’s net cash flows from operat-ing, investing, and financing activities to be positive or negative for its first three years of operations? Use the following format for your answers, and briefly explain your logic.

First Year Second Year Third YearNet cash fl ows from operating activities negativeNet cash fl ows from investing activitiesNet cash fl ows from fi nancing activities

Internet Project

Page 3: Cases & Projects

50 Chapter 1 Introduction to Accounting and Business50 Chapter 1 Introduction to Accounting and Business

CP 1-6 Financial analysis of Enron CorporationThe now defunct Enron Corporation, once headquartered in Houston, Texas, provided products and services for natural gas, electricity, and communications to wholesale and retail customers. Enron’s operations were conducted through a variety of subsidiaries and affiliates that involved transporting gas through pipelines, transmitting electricity, and managing energy commodities. The following data was taken from Enron’s financial statements:

In millionsTotal revenues $100,789Total costs and expenses 98,836Operating income 1,953Net income 979

Total assets 65,503Total liabilities 54,033Total stockholders’ equity 11,470

Net cash fl ows from operating activities 4,779Net cash fl ows from investing activities (4,264)Net cash fl ows from fi nancing activities 571Net increase in cash 1,086

The market price of Enron’s stock was approximately $83 per share when the prior fi-nancial statement data was taken. Before it went bankrupt, Enron’s stock sold for $0.22 per share.

Review the preceding financial statement data and search the Internet for articles on Enron Corporation. Briefly explain why Enron’s stock dropped so dramatically.

Internet Project

Page 4: Cases & Projects

100 Chapter 2 Analyzing Transactions

Enter the following transactions on Page 2 of the two-column journal.

July 16. Received $2,100 for serving as a disc jockey for a wedding reception.

18. Purchased supplies on account, $1,080.

21. Paid $620 to Upload Music for use of its current music demos in making vari-ous music sets.

22. Paid $800 to a local radio station to advertise the services of PS Music twice daily for the remainder of July.

23. Served as disc jockey for a party for $2,500. Received $750, with the remain-der due August 4, 2012.

27. Paid electric bill, $760.

28. Paid wages of $1,200 to receptionist and part-time assistant.

29. Paid miscellaneous expenses, $370.

30. Served as a disc jockey for a charity ball for $1,800. Received $400, with the remainder due on August 9, 2012.

31. Received $2,800 for serving as a disc jockey for a party.

31. Paid $1,400 royalties (music expense) to National Music Clearing for use of various artists’ music during July.

31. Paid dividends of $1,500.

PS Music’s chart of accounts and the balance of accounts as of July 1, 2012 (all normal balances), are as follows:

11 Cash $5,310 41 Fees Earned $6,65012 Accounts Receivable 1,250 50 Wages Expense 40014 Supplies 170 51 Offi ce Rent Expense 75015 Prepaid Insurance — 52 Equipment Rent Expense 70017 Offi ce Equipment — 53 Utilities Expense 30021 Accounts Payable 250 54 Music Expense 1,59023 Unearned Revenue — 55 Advertising Expense 45031 Capital Stock 5,000 56 Supplies Expense 18033 Dividends 500 59 Miscellaneous Expense 300

Instructions1. Enter the July 1, 2012, account balances in the appropriate balance column of a four-

column account. Write Balance in the Item column, and place a check mark ( ) in the Posting Reference column. (Hint: Verify the equality of the debit and credit balances in the ledger before proceeding with the next instruction.)

2. Analyze and journalize each transaction in a two-column journal beginning on Page 1, omitting journal entry explanations.

3. Post the journal to the ledger, extending the account balance to the appropriate bal-ance column after each posting.

4. Prepare an unadjusted trial balance as of July 31, 2012.

Cases & Projects

CP 2-1 Ethics and professional conduct in businessAt the end of the current month, Jonni Rembert prepared a trial balance for Star Rescue Service. The credit side of the trial balance exceeds the debit side by a significant amount. Jonni has decided to add the difference to the balance of the miscellaneous expense ac-count in order to complete the preparation of the current month’s financial statements by a 5 o’clock deadline. Jonni will look for the difference next week when she has more time.

Discuss whether Jonni is behaving in a professional manner.

amazzaro
Text Box
Chapter 2
Page 5: Cases & Projects

Chapter 2 Analyzing Transactions 101

CP 2-2 Account for revenueTucson College requires students to pay tuition each term before classes begin. Students who have not paid their tuition are not allowed to enroll or to attend classes.

What journal entry do you think Tucson College would use to record the receipt of the students’ tuition payments? Describe the nature of each account in the entry.

CP 2-3 Record transactionsThe following discussion took place between Erin Carr, the office manager of Panda Data Company, and a new accountant, Mark Goodell.

Mark: I’ve been thinking about our method of recording entries. It seems that it’s ineffi cient.

Erin: In what way?

Mark: Well—correct me if I’m wrong—it seems like we have unnecessary steps in the process. We could easily develop a trial balance by posting our transactions directly into the ledger and bypassing the journal altogether. In this way, we could combine the recording and posting process into one step and save ourselves a lot of time. What do you think?

Erin: We need to have a talk.

What should Erin say to Mark?

CP 2-4 Debits and credits

Group ProjectThe following excerpt is from a conversation between Boris Harris, the president and chief operating officer of Chesapeake Company, and his neighbor, Neil Liven.

Neil: Boris, I’m taking a course in night school, “Intro to Accounting.” I was wondering—could you answer a couple of questions for me?

Boris: Well, I will if I can.

Neil: Okay, our instructor says that it’s critical we understand the basic concepts of accounting, or we’ll never get beyond the fi rst test. My problem is with those rules of debit and credit . . . you know, assets increase with debits, decrease with credits, etc.

Boris: Yes, pretty basic stuff . You just have to memorize the rules. It shouldn’t be too diffi cult.

Neil: Sure, I can memorize the rules, but my problem is I want to be sure I understand the basic concepts be-hind the rules. For example, why can’t assets be increased with credits and decreased with debits like revenue? As long as everyone did it that way, why not? It would seem easier if we had the same rules for all increases and decreases in accounts. Also, why is the left side of an account called the debit side? Why couldn’t it be called something simple . . . like the “LE” for Left Entry? The right side could be called just “RE” for Right Entry. Finally, why are there just two sides to an entry? Why can’t there be three or four sides to an entry?

In a group of four or five, select one person to play the role of Boris and one person to play the role of Neil.

1. After listening to the conversation between Boris and Neil, help Boris answer Neil’s questions.

2. What information (other than just debit and credit journal entries) could the ac-counting system gather that might be useful to Boris in managing Chesapeake Company?

CP 2-5 Transactions and income statementAnwar Askari is planning to manage and operate AA Caddy Service at Mission Valley Golf and Country Club during June through August 2012. Anwar will rent a small maintenance building from the country club for $700 per month and will offer caddy services, including cart rentals, to golfers. Anwar has had no formal training in record keeping.

Anwar keeps notes of all receipts and expenses in a shoe box. An examination of Anwar’s shoe box records for June revealed the following:

Page 6: Cases & Projects

102 Chapter 2 Analyzing Transactions

June 1. Transferred $3,500 from personal bank account to be used to operate the caddy service.

1. Paid rent expense to Mission Valley Golf and Country Club, $700.

2. Paid for golf supplies (practice balls, etc.), $800.

3. Arranged for the rental of 25 regular (pulling) golf carts and 10 gasoline-driven carts for $3,000 per month. Paid $500 in advance, with the remaining $2,500 due June 20.

7. Purchased supplies, including gasoline, for the golf carts on account, $600. Mission Valley Golf and Country Club has agreed to allow Anwar to store the gasoline in one of its fuel tanks at no cost.

15. Received cash for services from June 1–15, $4,150.

17. Paid cash to creditors on account, $600.

20. Paid remaining rental on golf carts, $2,500.

22. Purchased supplies, including gasoline, on account, $400.

25. Accepted IOUs from customers on account, $1,800.

28. Paid miscellaneous expenses, $350.

30. Received cash for services from June 16–30, $6,350.

30. Paid telephone and electricity (utilities) expenses, $340.

30. Paid wages of part-time employees, $850.

30. Received cash in payment of IOUs on account, $1,200.

30. Determined the amount of supplies on hand at the end of June, $500.

Anwar has asked you several questions concerning his financial affairs to date, and he has asked you to assist with his record keeping and reporting of financial data.

a. To assist Anwar with his record keeping, prepare a chart of accounts that would be appropriate for AA Caddy Service. Note: Small businesses such as Ace Caddy Service are often organized as proprietorships. The accounting for proprietorships is similar to that for a corporation, except that the owner’s equity accounts differ. Specifically, instead of the account for Capital Stock, a capital account entitled Anwar Askari, Capital is used to record investments in the business. In addition, instead of a dividends account, withdrawals from the business are debited to Anwar Askari, Drawing. A proprietorship has no retained earnings account.

b. Prepare an income statement for June in order to help Anwar assess the profitability of AA Caddy Service. For this purpose, the use of T accounts may be helpful in ana-lyzing the effects of each June transaction.

c. Based on Anwar’s records of receipts and payments, compute the amount of cash on hand on June 30. For this purpose, a T account for cash may be useful.

d. A count of the cash on hand on June 30 totaled $8,390. Briefly discuss the possible causes of the difference between the amount of cash computed in (c) and the actual amount of cash on hand.

CP 2-6 Opportunities for accountantsThe increasing complexity of the current business and regulatory environment has created an increased demand for accountants who can analyze business transactions and interpret their effects on the financial statements. In addition, a basic ability to analyze the effects of transactions is necessary to be successful in all fields of business as well as in other disciplines, such as law. To better understand the importance of accounting in today’s environment, search the Internet or your local newspaper for job opportunities. One possible Internet site is http://www.careerbuilder.com. Then do one of the following:

1. Print a listing of one or two ads for accounting jobs. Alternatively, bring to class one or two newspaper ads for accounting jobs.

2. Print a listing of one or two ads for nonaccounting jobs for which some knowledge of accounting is preferred or necessary. Alternatively, bring to class one or two news-paper ads for such jobs.

Internet Project

Page 7: Cases & Projects

146 Chapter 3 The Adjusting Process

PS MusicUnadjusted Trial Balance

July 31, 2012Debit

BalancesCredit

Balances

Cash . . . . . . . . . . . . . . . . . . . . . . . . . . . . . . . . . . . . . . . . . . . . . . . . . . . . . . . . . . . . . . . . . 10,510Accounts Receivable. . . . . . . . . . . . . . . . . . . . . . . . . . . . . . . . . . . . . . . . . . . . . . . . . . . 3,150Supplies . . . . . . . . . . . . . . . . . . . . . . . . . . . . . . . . . . . . . . . . . . . . . . . . . . . . . . . . . . . . . . 1,250Prepaid Insurance . . . . . . . . . . . . . . . . . . . . . . . . . . . . . . . . . . . . . . . . . . . . . . . . . . . . . 2,700Office Equipment . . . . . . . . . . . . . . . . . . . . . . . . . . . . . . . . . . . . . . . . . . . . . . . . . . . . . 6,000Accounts Payable . . . . . . . . . . . . . . . . . . . . . . . . . . . . . . . . . . . . . . . . . . . . . . . . . . . . . 7,080Unearned Revenue . . . . . . . . . . . . . . . . . . . . . . . . . . . . . . . . . . . . . . . . . . . . . . . . . . . . 7,200Capital Stock. . . . . . . . . . . . . . . . . . . . . . . . . . . . . . . . . . . . . . . . . . . . . . . . . . . . . . . . . . 9,000Dividends. . . . . . . . . . . . . . . . . . . . . . . . . . . . . . . . . . . . . . . . . . . . . . . . . . . . . . . . . . . . . 2,000Fees Earned. . . . . . . . . . . . . . . . . . . . . . . . . . . . . . . . . . . . . . . . . . . . . . . . . . . . . . . . . . . 16,750Music Expense . . . . . . . . . . . . . . . . . . . . . . . . . . . . . . . . . . . . . . . . . . . . . . . . . . . . . . . . 3,610Wages Expense . . . . . . . . . . . . . . . . . . . . . . . . . . . . . . . . . . . . . . . . . . . . . . . . . . . . . . . 2,800Office Rent Expense . . . . . . . . . . . . . . . . . . . . . . . . . . . . . . . . . . . . . . . . . . . . . . . . . . . 2,550Advertising Expense . . . . . . . . . . . . . . . . . . . . . . . . . . . . . . . . . . . . . . . . . . . . . . . . . . . 1,450Equipment Rent Expense . . . . . . . . . . . . . . . . . . . . . . . . . . . . . . . . . . . . . . . . . . . . . . 1,300Utilities Expense . . . . . . . . . . . . . . . . . . . . . . . . . . . . . . . . . . . . . . . . . . . . . . . . . . . . . . 1,060Supplies Expense. . . . . . . . . . . . . . . . . . . . . . . . . . . . . . . . . . . . . . . . . . . . . . . . . . . . . . 180Miscellaneous Expense . . . . . . . . . . . . . . . . . . . . . . . . . . . . . . . . . . . . . . . . . . . . . . . . 1,470

40,030 40,030

The data needed to determine adjustments for the two-month period ending July 31, 2012, are as follows:

a. During July, PS Music provided guest disc jockeys for WHBD for a total of 120 hours. For information on the amount of the accrued revenue to be billed to WHBD, see the contract described in the July 3, 2012, transaction at the end of Chapter 2.

b. Supplies on hand at July 31, $400.

c. The balance of the prepaid insurance account relates to the July 1, 2012, transaction at the end of Chapter 2.

d. Depreciation of the office equipment is $75.

e. The balance of the unearned revenue account relates to the contract between PS Music and WHBD, described in the July 3, 2012, transaction at the end of Chapter 2.

f. Accrued wages as of July 31, 2012, were $170.

Instructions1. Prepare adjusting journal entries. You will need the following additional accounts:

18 Accumulated Depreciation—Office Equipment

22 Wages Payable

57 Insurance Expense

58 Depreciation Expense

2. Post the adjusting entries, inserting balances in the accounts affected.

3. Prepare an adjusted trial balance.

CP 3-1 Ethics and professional conduct in businessJoshua Thorp opened Laser Co. on January 1, 2011. At the end of the first year, the busi-ness needed additional funds. On behalf of Laser, Joshua applied to Vermont National Bank for a loan of $500,000. Based on Laser financial statements, which had been prepared on a cash basis, the Vermont National Bank loan officer rejected the loan as too risky.

After receiving the rejection notice, Joshua instructed his accountant to prepare the financial statements on an accrual basis. These statements included $90,000 in accounts

Cases & Projects

amazzaro
Text Box
Chapter 3
Page 8: Cases & Projects

Chapter 3 The Adjusting Process 147

receivable and $35,000 in accounts payable. Joshua then instructed his accountant to record an additional $25,000 of accounts receivable for commissions on property for which a contract had been signed on December 28, 2011. The title to the property is to transfer on January 5, 2012, when an attorney formally records the transfer of the property to the buyer.

Joshua then applied for a $500,000 loan from NYC Bank, using the revised financial state-ments. On this application, Joshua indicated that he had not previously been rejected for credit.

Discuss the ethical and professional conduct of Joshua Thorp in applying for the loan from NYC Bank.

CP 3-2 Accrued expenseOn December 30, 2012, you buy a Ford 350F truck. It comes with a three-year, 48,000-mile warranty. On March 5, 2013, you return the truck to the dealership for some basic repairs covered under the warranty. The cost of the repairs to the dealership is $2,400. Assume that based upon past history, Ford Motor Company can reasonably estimate the cost of repairs for each model year for its Ford 350F. In what year, 2012 or 2013, should Ford recognize the cost of the warranty repairs as an expense?

CP 3-3 Accrued revenueThe following is an excerpt from a conversation between Kay Scott and Jeff Lee just before they boarded a flight to London on Delta Air Lines. They are going to London to attend their company’s annual sales conference.

Kay: Jeff , aren’t you taking an introductory accounting course at college?

Jeff : Yes, I decided it’s about time I learned something about accounting. You know, our annual bonuses are based on the sales fi gures that come from the accounting department.

Kay: I guess I never really thought about it.

Jeff : You should think about it! Last year, I placed a $1,000,000 order on December 28. But when I got my bonus, the $1,000,000 sale wasn’t included. They said it hadn’t been shipped until January 5, so it would have to count in next year’s bonus.

Kay: A real bummer!

Jeff : Right! I was counting on that bonus including the $1,000,000 sale.

Kay: Did you complain?

Jeff : Yes, but it didn’t do any good. Lori, the head accountant, said something about matching revenues and ex-penses. Also, something about not recording revenues until the sale is fi nal. I fi gure I’d take the accounting course and fi nd out whether she’s just messing with me.

Kay: I never really thought about it. When do you think Delta Air Lines will record its revenues from this fl ight?

Jeff : Hmmm . . . I guess it could record the revenue when it sells the ticket . . . or . . . when the boarding passes are scanned at the door . . . or . . . when we get off the plane . . . or when our company pays for the tickets . . . or . . . I don’t know. I’ll ask my accounting instructor.

Discuss when Delta Air Lines should recognize the revenue from ticket sales to properly match revenues and expenses.

CP 3-4 Adjustments and financial statementsSeveral years ago, your brother opened Granite Appliance Repairs. He made a small initial investment and added money from his personal bank account as needed. He withdrew money for living expenses at irregular intervals. As the business grew, he hired an as-sistant. He is now considering adding more employees, purchasing additional service trucks, and purchasing the building he now rents. To secure funds for the expansion, your brother submitted a loan application to the bank and included the most recent financial statements (shown below) prepared from accounts maintained by a part-time bookkeeper.

Page 9: Cases & Projects

148 Chapter 3 The Adjusting Process

Granite Appliance RepairsIncome Statement

For the Year Ended July 31, 2012

Service revenue . . . . . . . . . . . . . . . . . $225,000Less: Rent paid . . . . . . . . . . . . . . . . . . $62,400

Wages paid . . . . . . . . . . . . . . . . 49,500Supplies paid. . . . . . . . . . . . . . . 14,000Utilities paid . . . . . . . . . . . . . . . 13,000Insurance paid . . . . . . . . . . . . . 7,200Miscellaneous payments . . . . 18,200 164,300

Net income . . . . . . . . . . . . . . . . . . . . . $ 60,700Granite Appliance Repairs

Balance SheetJuly 31, 2012

AssetsCash . . . . . . . . . . . . . . . . . . . . . . . . . . . $ 31,800Amounts due from customers . . . . 37,500Truck . . . . . . . . . . . . . . . . . . . . . . . . . . . 110,700Total assets . . . . . . . . . . . . . . . . . . . . . $180,000

EquitiesOwner’s equity . . . . . . . . . . . . . . . . . . $180,000

After reviewing the financial statements, the loan officer at the bank asked your brother if he used the accrual basis of accounting for revenues and expenses. Your brother responded that he did and that is why he included an account for “Amounts Due from Custom-ers.” The loan officer then asked whether or not the accounts were adjusted prior to the preparation of the statements. Your brother answered that they had not been adjusted.

a. Why do you think the loan officer suspected that the accounts had not been adjusted prior to the preparation of the statements?

b. Indicate possible accounts that might need to be adjusted before an accurate set of financial statements could be prepared.

CP 3-5 Codes of ethicsObtain a copy of your college or university’s student code of conduct. In groups of three or four, answer the following questions:

1. Compare this code of conduct with the accountant’s Codes of Professional Conduct, which is linked to the text Web site at www.cengage.com/accounting/warren.

2. One of your classmates asks you for permission to copy your homework, which your instructor will be collecting and grading for part of your overall term grade. Although your instructor has not stated whether one student may or may not copy another stu-dent’s homework, is it ethical for you to allow your classmate to copy your homework? Is it ethical for your classmate to copy your homework?

Page 10: Cases & Projects

Chapter 4 Completing the Accounting Cycle 209

d. Accrued receptionist salary on May 31 is $300.

e. Rent expired during May is $1,600.

f. Unearned fees on May 31 are $1,500.

5. Optional: Enter the unadjusted trial balance on an end-of-period spreadsheet (work sheet) and complete the spreadsheet.

6. Journalize and post the adjusting entries. Record the adjusting entries on Page 7 of the journal.

7. Prepare an adjusted trial balance.

8. Prepare an income statement, a retained earnings statement, and a balance sheet.

9. Prepare and post the closing entries. Record the closing entries on Page 8 of the journal. (Income Summary is account #34 in the chart of accounts.) Indicate closed accounts by inserting a line in both the Balance columns opposite the closing entry.

10. Prepare a post-closing trial balance.

Cases & Projects

CP 4-1 Ethics and professional conduct in businessLaser Graphics is a graphics arts design consulting firm. Spencer Lowry, its treasurer and vice president of finance, has prepared a classified balance sheet as of March 31, 2012, the end of its fiscal year. This balance sheet will be submitted with Laser Graphics’ loan application to American Trust & Savings Bank.

In the Current Assets section of the balance sheet, Spencer reported a $90,000 receivable from Jackie Doyle, the president of Laser Graphics, as a trade account receivable. Jackie bor-rowed the money from Laser Graphics in April 2010 for a down payment on a new home. She has orally assured Spencer that she will pay off the account receivable within the next year. Spencer reported the $90,000 in the same manner on the preceding year’s balance sheet.

Evaluate whether it is acceptable for Spencer to prepare the March 31, 2012, balance sheet in the manner indicated above.

CP 4-2 Financial statementsThe following is an excerpt from a telephone conversation between Jared Bodine, presi-dent of Hometown Supplies Co., and Julie Sims, owner of Express Employment Co.Jared: Julie, you’re going to have to do a better job of fi nding me a new computer programmer. That last guy was great at programming, but he didn’t have any common sense.

Julie: What do you mean? The guy had a master’s degree with straight A’s.

Jared: Yes, well, last month he developed a new fi nancial reporting system. He said we could do away with manually preparing an end-of-period spreadsheet (work sheet) and fi nancial statements. The computer would automatically generate our fi nancial statements with “a push of a button.”

Julie: So what’s the big deal? Sounds to me like it would save you time and eff ort.

Jared: Right! The balance sheet showed a minus for supplies!

Julie: Minus supplies? How can that be?

Jared: That’s what I asked.

Julie: So, what did he say?

Jared: Well, after he checked the program, he said that it must be right. The minuses were greater than the pluses. . . .

Julie: Didn’t he know that Supplies can’t have a credit balance—it must have a debit balance?

Jared: He asked me what a debit and credit were.

Julie: I see your point.

1. Comment on (a) the desirability of computerizing Hometown Supplies Co.’s financial reporting system, (b) the elimination of the end-of-period spreadsheet (work sheet) in a computerized accounting system, and (c) the computer programmer’s lack of accounting knowledge.

2. Explain to the programmer why Supplies could not have a credit balance.

amazzaro
Text Box
Chapter 4
Page 11: Cases & Projects

210 Chapter 4 Completing the Accounting Cycle

CP 4-3 Financial statementsAssume that you recently accepted a position with Frontier National Bank as an assistant loan officer. As one of your first duties, you have been assigned the responsibility of evaluating a loan request for $150,000 from Icancreateart.com, a small corporation. In support of the loan application, Tess Ramey, owner and sole stockholder, submitted a “Statement of Accounts” (trial balance) for the first year of operations ended July 31, 2012.

Icancreateart.comStatement of Accounts

July 31, 2012Cash . . . . . . . . . . . . . . . . . . . . . . . . . . . . . . . . . . . . . . . . . . . . . . . . . . . . . . . . . . . . . 5,000Billings Due from Others . . . . . . . . . . . . . . . . . . . . . . . . . . . . . . . . . . . . . . . . . . . 40,000Supplies (chemicals, etc.) . . . . . . . . . . . . . . . . . . . . . . . . . . . . . . . . . . . . . . . . . . 7,500Building . . . . . . . . . . . . . . . . . . . . . . . . . . . . . . . . . . . . . . . . . . . . . . . . . . . . . . . . . . 122,300Equipment. . . . . . . . . . . . . . . . . . . . . . . . . . . . . . . . . . . . . . . . . . . . . . . . . . . . . . . . 25,000Amounts Owed to Others . . . . . . . . . . . . . . . . . . . . . . . . . . . . . . . . . . . . . . . . . . 11,000Investment in Business . . . . . . . . . . . . . . . . . . . . . . . . . . . . . . . . . . . . . . . . . . . . 74,000Service Revenue . . . . . . . . . . . . . . . . . . . . . . . . . . . . . . . . . . . . . . . . . . . . . . . . . . 215,000Wages Expense . . . . . . . . . . . . . . . . . . . . . . . . . . . . . . . . . . . . . . . . . . . . . . . . . . . 75,000Utilities Expense . . . . . . . . . . . . . . . . . . . . . . . . . . . . . . . . . . . . . . . . . . . . . . . . . . 10,000Rent Expense . . . . . . . . . . . . . . . . . . . . . . . . . . . . . . . . . . . . . . . . . . . . . . . . . . . . . 8,000Insurance Expense . . . . . . . . . . . . . . . . . . . . . . . . . . . . . . . . . . . . . . . . . . . . . . . . 6,000Other Expenses . . . . . . . . . . . . . . . . . . . . . . . . . . . . . . . . . . . . . . . . . . . . . . . . . . . 1,200

300,000 300,000

1. Explain to Tess Ramey why a set of financial statements (income statement, retained earnings statement, and balance sheet) would be useful to you in evaluating the loan request.

2. In discussing the “Statement of Accounts” with Tess Ramey, you discovered that the accounts had not been adjusted at July 31. Analyze the “Statement of Accounts” and indicate possible adjusting entries that might be necessary before an accurate set of financial statements could be prepared.

3. Assuming that an accurate set of financial statements will be submitted by Tess Ramey in a few days, what other considerations or information would you require before making a decision on the loan request?

CP 4-4 Compare balance sheets

Group ProjectIn groups of three or four, compare the balance sheets of two different companies, and present to the class a summary of the similarities and differences of the two companies. You may obtain the balance sheets you need from one of the following sources:

1. Your school or local library.

2. The investor relations department of each company.

3. The company’s Web site on the Internet.

4. EDGAR (Electronic Data Gathering, Analysis, and Retrieval), the electronic archives of financial statements filed with the Securities and Exchange Commission.

SEC documents can be retrieved using the EdgarScan™ service at http://sec.gov. To obtain annual report information, click on “Search for Company Filing,” click on “Companies & Other Filers,” type in the company name, and then click on “Find Companies.” Click on the CIK related to the company name, search for Form 10-K, and click on “Retrieve Selected Findings.” Finally, click on the “html” for the latest period and the related document.

Internet Project

Page 12: Cases & Projects

Chapter 5 Accounting for Merchandising Businesses 265

July 24. Returned $5,000 of damaged merchandise purchased on July 21, receiving credit from the seller.

26. Refunded cash on sales made for cash, $12,000. The cost of the merchandise returned was $7,200.

28. Paid sales salaries of $22,800 and offi ce salaries of $15,200.

29. Purchased store supplies for cash, $2,400.

30. Sold merchandise on account to Dix Co., terms 2/10, n/30, FOB shipping point, $18,750. The cost of the merchandise sold was $11,250.

30. Received cash from sale of July 20, less discount, plus freight paid on July 21.

31. Paid for purchase of July 21, less return of July 24 and discount.

Instructions 1. Enter the balances of each of the accounts in the appropriate balance column of a

four-column account. Write Balance in the item section, and place a check mark ( ) in the Posting Reference column. Journalize the transactions for July starting on page 20 of the journal.

2. Post the journal to the general ledger, extending the month-end balances to the appropriate balance columns after all posting is completed. In this problem, you are not required to update or post to the accounts receivable and accounts payable subsidiary ledgers.

3. Prepare an unadjusted trial balance.

4. At the end of July, the following adjustment data were assembled. Analyze and use these data to complete (5) and (6).

a. Merchandise inventory on July 31 $565,000b. Insurance expired during the year 13,400c. Store supplies on hand on July 31 3,900d. Depreciation for the current year 11,500e. Accrued salaries on July 31:

Sales salaries $3,200Offi ce salaries 1,300 4,500

5. Optional: Enter the unadjusted trial balance on a 10-column end-of-period spreadsheet (work sheet), and complete the spreadsheet.

6. Journalize and post the adjusting entries. Record the adjusting entries on page 22 of the journal.

7. Prepare an adjusted trial balance.

8. Prepare an income statement, a retained earnings statement, and a balance sheet.

9. Prepare and post the closing entries. Record the closing entries on page 23 of the journal. Indicate closed accounts by inserting a line in both the Balance columns opposite the closing entry. Insert the new balance in the retained earnings account.

10. Prepare a post-closing trial balance.

Cases & Projects

CP 5-1 Ethics and professional conduct in businessOn March 13, 2012, Plant-Wise Company, a garden retailer, purchased $18,000 of seed, terms 2/10, n/30, from Premium Seed Co. Even though the discount period had expired, Brendan Morton subtracted the discount of $360 when he processed the documents for payment on March 26, 2012.

Discuss whether Brendan Morton behaved in a professional manner by subtract-ing the discount, even though the discount period had expired.

amazzaro
Text Box
Chapter 5
Page 13: Cases & Projects

266 Chapter 5 Accounting for Merchandising Businesses

CP 5-2 Purchases discounts and accounts payableBud’s Video Store Co. is owned and operated by Jim Budeski. The following is an excerpt from a conversation between Jim Budeski and Ann Pavik, the chief accountant for Bud’s Video Store.

Jim: Ann, I’ve got a question about this recent balance sheet.

Ann: Sure, what’s your question?

Jim: Well, as you know, I’m applying for a bank loan to fi nance our new store in Coronado, and I noticed that the accounts payable are listed as $235,000.

Ann: That’s right. Approximately $190,000 of that represents amounts due our suppliers, and the remainder is miscellaneous payables to creditors for utilities, offi ce equipment, supplies, etc.

Jim: That’s what I thought. But as you know, we normally receive a 2% discount from our suppliers for earlier payment, and we always try to take the discount.

Ann: That’s right. I can’t remember the last time we missed a discount.

Jim: Well, in that case, it seems to me the accounts payable should be listed minus the 2% discount. Let’s list the accounts payable due suppliers as $186,200, rather than $190,000. Every little bit helps. You never know. It might make the diff erence between getting the loan and not.

How would you respond to Jim Budeski’s request?

CP 5-3 Determining cost of purchaseThe following is an excerpt from a conversation between Jon Akers and Deb Flack. Jon is debating whether to buy a stereo system from Old Town Audio, a locally owned electronics store, or Sound Pro, an online electronics company.

Jon: Deb, I don’t know what to do about buying my new stereo.

Deb: What’s the problem?

Jon: Well, I can buy it locally at Old Town Audio for $1,400.00. However, Sound Pro has the same system listed for $1,150.00.

Deb: So what’s the big deal? Buy it from Sound Pro.

Jon: It’s not quite that simple. Sound Pro charges $39.99 for shipping and handling. If I have them send it next-day air, it’ll cost $69.99 for shipping and handling.

Deb: I guess it is a little confusing.

Jon: That’s not all. Old Town Audio will give an additional 2% discount if I pay cash. Otherwise, they will let me use my VISA, or I can pay it off in three monthly installments. In addition, if I buy it from Old Town Audio I have to pay 8% sales tax. I won’t have to pay sales tax if I buy it from Sound Pro since they are out of state.

Deb: Anything else???

Jon: Well … Sound Pro says I have to charge it on my VISA. They don’t accept checks.

Deb: I am not surprised. Many online stores don’t accept checks.

Jon: I give up. What would you do?

1. Assuming that Sound Pro doesn’t charge sales tax on the sale to Jon, which company is offering the best buy?

2. What might be some considerations other than price that might influence Jon’s decision on where to buy the stereo system?

CP 5-4 Sales discountsYour sister operates Budget Parts Company, an online boat parts distributorship that is in its third year of operation. The income statement is shown below and was recently prepared for the year ended July 31, 2012.

Page 14: Cases & Projects

Chapter 5 Accounting for Merchandising Businesses 267

Budget Parts CompanyIncome Statement

For the Year Ended July 31, 2012Revenues:

Net sales . . . . . . . . . . . . . . . . . . . . . . . . . . . . . . . . . . . . . . . . . . . . . . . . . . . . . . . . $600,000Interest revenue . . . . . . . . . . . . . . . . . . . . . . . . . . . . . . . . . . . . . . . . . . . . . . . . . 7,500

Total revenues. . . . . . . . . . . . . . . . . . . . . . . . . . . . . . . . . . . . . . . . . . . . . . . . $607,500Expenses:

Cost of merchandise sold. . . . . . . . . . . . . . . . . . . . . . . . . . . . . . . . . . . . . . . . . $360,000Selling expenses . . . . . . . . . . . . . . . . . . . . . . . . . . . . . . . . . . . . . . . . . . . . . . . . . 67,500Administrative expenses . . . . . . . . . . . . . . . . . . . . . . . . . . . . . . . . . . . . . . . . . 36,000Interest expense . . . . . . . . . . . . . . . . . . . . . . . . . . . . . . . . . . . . . . . . . . . . . . . . . 11,250

Total expenses . . . . . . . . . . . . . . . . . . . . . . . . . . . . . . . . . . . . . . . . . . . . . . . 474,750Net income . . . . . . . . . . . . . . . . . . . . . . . . . . . . . . . . . . . . . . . . . . . . . . . . . . . . . . . $132,750

Your sister is considering a proposal to increase net income by offering sales discounts of 2/15, n/30, and by shipping all merchandise FOB shipping point. Currently, no sales discounts are allowed and merchandise is shipped FOB destination. It is estimated that these credit terms will increase net sales by 15%. The ratio of the cost of merchandise sold to net sales is expected to be 60%. All selling and administrative expenses are expected to remain unchanged, except for store supplies, miscellaneous selling, office supplies, and miscellaneous administrative expenses, which are expected to increase proportionately with increased net sales. The amounts of these preceding items for the year ended July 31, 2012, were as follows:

Store supplies expense $9,000Miscellaneous selling expense 2,400Offi ce supplies expense 2,000Miscellaneous administrative expense 1,000

The other income and other expense items will remain unchanged. The shipment of all merchandise FOB shipping point will eliminate all delivery expenses, which for the year ended July 31, 2012, were $12,000.

1. Prepare a projected single-step income statement for the year ending July 31, 2013, based on the proposal. Assume all sales are collected within the discount period.

2. a. Based on the projected income statement in (1), would you recommend the implementation of the proposed changes?

b. Describe any possible concerns you may have related to the proposed changes described in (1).

CP 5-5 Shopping for a television

Group ProjectAssume that you are planning to purchase a 52-inch LCD, flat screen television. In groups of three or four, determine the lowest cost for the television, considering the available alternatives and the advantages and disadvantages of each alternative. For example, you could purchase locally, through mail order, or through an Internet shopping service. Consider such factors as delivery charges, interest-free financing, discounts, coupons, and availability of warranty services. Prepare a report for presentation to the class.

Page 15: Cases & Projects

Chapter 6 Inventories 309

1. Draw a line through the quantity, and insert the quantity and unit cost of the last purchase.

2. On the following line, insert the quantity and unit cost of the next-to-the-last purchase.

3. Total the cost and market columns and insert the lower of the two totals in the Lower of C or M column. The first item on the inventory sheet has been completed as an example.

Appendix PR 6-5B Retail method; gross profit methodSelected data on merchandise inventory, purchases, and sales for Segal Co. and Iroquois Co. are as follows:

Cost RetailSegal Co.Merchandise inventory, March 1 $ 298,000 $ 375,000Transactions during March:

Purchases (net) 4,850,000 6,225,000Sales 6,320,000Sales returns and allowances 245,000

Iroquois Co.Merchandise inventory, January 1 $ 300,000Transactions during January thru March:

Purchases (net) 4,150,000Sales 6,900,000Sales returns and allowances 175,000

Estimated gross profi t rate 40%

Instructions1. Determine the estimated cost of the merchandise inventory of Segal Co. on March 31

by the retail method, presenting details of the computations.

2. a. Estimate the cost of the merchandise inventory of Iroquois Co. on March 31 by the gross profit method, presenting details of the computations.

b. Assume that Iroquois Co. took a physical inventory on March 31 and discovered that $396,500 of merchandise was on hand. What was the estimated loss of inven-tory due to theft or damage during January thru March?

Cases & Projects

CP 6-1 Ethics and professional conduct in businessContours Co. is experiencing a decrease in sales and operating income for the fiscal year ending July 31, 2012. Mark Irwin, controller of Contours Co., has suggested that all orders received before the end of the fiscal year be shipped by midnight, July 31, 2012, even if the shipping department must work overtime. Since Contours Co. ships all merchandise FOB shipping point, it would record all such shipments as sales for the year ending July 31, 2012, thereby offsetting some of the decreases in sales and operating income.

Discuss whether Mark Irwin is behaving in a professional manner.

CP 6-2 LIFO and inventory flowThe following is an excerpt from a conversation between Gary Ortiz, the warehouse manager for Ivey Foods Wholesale Co., and its accountant, Lori Cray. Ivey Foods operates a large regional warehouse that supplies produce and other grocery products to grocery stores in smaller communities.

1. $409,500

amazzaro
Text Box
Chapter 6
Page 16: Cases & Projects

310 Chapter 6 Inventories

Gary: Lori, can you explain what’s going on here with these monthly statements?

Lori: Sure, Gary. How can I help you?

Gary: I don’t understand this last-in, fi rst-out inventory procedure. It just doesn’t make sense.

Lori: Well, what it means is that we assume that the last goods we receive are the fi rst ones sold. So the inventory consists of the items we purchased fi rst.

Gary: Yes, but that’s my problem. It doesn’t work that way! We always distribute the oldest produce fi rst. Some of that produce is perishable! We can’t keep any of it very long or it’ll spoil.

Lori: Gary, you don’t understand. We only assume that the products we distribute are the last ones received. We don’t actually have to distribute the goods in this way.

Gary: I always thought that accounting was supposed to show what really happened. It all sounds like “make be-lieve” to me! Why not report what really happens?

Respond to Gary’s concerns.

CP 6-3 Costing inventoryWhite Dove Company began operations in 2012 by selling a single product. Data on purchases and sales for the year were as follows:

Purchases:

Date Units Purchased Unit Cost Total CostApril 6 62,000 $12.20 $ 756,400May 18 66,000 13.00 858,000June 6 80,000 13.20 1,056,000July 10 80,000 14.00 1,120,000August 10 54,400 14.25 775,200October 25 25,600 14.50 371,200November 4 16,000 14.95 239,200December 10 16,000 16.00 256,000

400,000 $5,432,000

Sales:April 32,000 unitsMay 32,000June 40,000July 48,000August 56,000September 56,000October 36,000November 20,000December 16,000Total units 336,000Total sales $5,200,000

On January 4, 2013, the president of the company, Joel McLees, asked for your advice on costing the 64,000-unit physical inventory that was taken on December 31, 2012. Moreover, since the firm plans to expand its product line, he asked for your advice on the use of a perpetual inventory system in the future.

1. Determine the cost of the December 31, 2012, inventory under the periodic system, using the (a) first-in, first-out method, (b) last-in, first-out method, and (c) average cost method.

2. Determine the gross profit for the year under each of the three methods in (1).

3. a. Explain varying viewpoints why each of the three inventory costing meth-ods may best reflect the results of operations for 2012.

b. Which of the three inventory costing methods may best reflect the replace-ment cost of the inventory on the balance sheet as of December 31, 2012?

c. Which inventory costing method would you choose to use for income tax purposes? Why?

d. Discuss the advantages and disadvantages of using a perpetual inventory system. From the data presented in this case, is there any indication of the adequacy of inventory levels during the year?

Page 17: Cases & Projects

Chapter 6 Inventories 311

CP 6-4 Inventory ratios for Dell and HPDell Inc. and Hewlett-Packard Development Company, L.P. (HP) are both manufacturers of computer equipment and peripherals. However, the two companies follow two different strategies. Dell follows primarily a build-to-order strategy, where the consumer orders the computer from a Web page. The order is then manufactured and shipped to the customer within days of the order. In contrast, HP follows a build-to-stock strategy, where the computer is first built for inventory, then sold from inventory to retailers, such as Best Buy. The two strategies can be seen in the difference between the inventory turnover and number of days’ sales in inventory ratios for the two companies. The following financial statement information is provided for Dell and HP for a recent fiscal year (in millions):

Dell HPInventory, beginning of period $ 1,180 $ 7,879Inventory, end of period 867 6,128Cost of goods sold 50,144 87,524

a. Determine the inventory turnover ratio and number of days’ sales in inventory ratio for each company. Round to one decimal place.

b. Interpret the difference between the ratios for the two companies.

CP 6-5 Comparing inventory ratios for two companiesTiffany Co. is a high-end jewelry retailer, while Amazon.com uses its e-commerce services, features, and technologies to sell its products through the Internet. Recent balance sheet inventory disclosures for Tiffany and Amazon.com (in millons) are as follows:

End-of-Period Inventory Beginning-of-Period InventoryTiff any Co. $1,601 $1,242Amazon.com 1,399 1,200

The cost of merchandise sold reported by each company was as follows:

Tiff any Co. Amazon.comCost of merchandise sold $1,215 $14,896

a. Determine the inventory turnover and number of days’ sales in inventory for Tiffany and Amazon.com. Round to two decimal places and nearest day.

b. Interpret your results.

CP 6-6 Comparing inventory ratios for three companiesThe general merchandise retail industry has a number of segments represented by the following companies:

Company Name Merchandise ConceptCostco Wholesale Corporation Membership warehouseWal-Mart Discount general merchandiseJCPenney Department store

For a recent year, the following cost of merchandise sold and beginning and ending inventories have been provided from corporate annual reports (in millions) for these three companies:

Costco Wal-Mart JCPenneyCost of merchandise sold $62,335 $306,158 $11,571Merchandise inventory, beginning 5,039 35,180 3,641Merchandise inventory, ending 5,405 34,511 3,259

a. Determine the inventory turnover ratio for all three companies. Round to one decimal place.

b. Determine the number of days’ sales in inventory for all three companies. Round to one decimal place.

c. Interpret these results based on each company’s merchandise concept.

F•A•I

F•A•I

F•A•I

Page 18: Cases & Projects

Chapter 7 Sarbanes-Oxley, Internal Control, and Cash 355

CP 7-1 Ethics and professional conduct in businessDuring the preparation of the bank reconciliation for Regal Concepts Co., Misty Watts, the assistant controller, discovered that Windsor National Bank incorrectly recorded a $15,750 check written by Regal Concepts Co. as $1,575. Misty has decided not to notify the bank but wait for the bank to detect the error. Misty plans to record the $14,175 error as Other Income if the bank fails to detect the error within the next three months.

Discuss whether Misty is behaving in a professional manner.

CP 7-2 Internal controlsThe following is an excerpt from a conversation between two sales clerks, Craig Rice and Jill Allen. Craig and Jill are employed by Ogden Electronics, a locally owned and operated electronics retail store.

Craig: Did you hear the news?

Jill: What news?

Craig: Kate and Steve were both arrested this morning.

Jill: What? Arrested? You’re putting me on!

Craig: No, really! The police arrested them fi rst thing this morning. Put them in handcuff s, read them their rights—the whole works. It was unreal!

Jill: What did they do?

Craig: Well, apparently they were fi lling out merchandise refund forms for fi ctitious customers and then taking the cash.

Jill: I guess I never thought of that. How did they catch them?

Craig: The store manager noticed that returns were twice that of last year and seemed to be increasing. When he confronted Kate, she became fl ustered and admitted to taking the cash, apparently over $10,000 in just three months. They’re going over the last six months’ transactions to try to determine how much Steve stole. He appar-ently started stealing fi rst.

Suggest appropriate control procedures that would have prevented or detected the theft of cash.

CP 7-3 Internal controlsThe following is an excerpt from a conversation between the store manager of Dozier Brothers Grocery Stores, Amy Blankenship, and Mike Ulrich, president of Dozier Broth-ers Grocery Stores.

Mike: Amy, I’m concerned about this new scanning system.

Amy: What’s the problem?

Mike: Well, how do we know the clerks are ringing up all the merchandise?

Amy: That’s one of the strong points about the system. The scanner automatically rings up each item, based on its bar code. We update the prices daily, so we’re sure that the sale is rung up for the right price.

Mike: That’s not my concern. What keeps a clerk from pretending to scan items and then simply not charging his friends? If his friends were buying 10-15 items, it would be easy for the clerk to pass through several items with his fi nger over the bar code or just pass the merchandise through the scanner with the wrong side showing. It would look normal for anyone observing. In the old days, we at least could hear the cash register ringing up each sale.

Amy: I see your point.

Suggest ways that Dozier Brothers Grocery Stores could prevent or detect the theft of merchandise as described.

Cases & Projects

amazzaro
Text Box
Chapter 7
Page 19: Cases & Projects

356 Chapter 7 Sarbanes-Oxley, Internal Control, and Cash

CP 7-4 Ethics and professional conduct in businessEric Inman and Darcy Getz are both cash register clerks for Farmer John’s Markets. Nancy McNeil is the store manager for Farmer John’s Markets. The following is an excerpt of a conversation between Eric and Darcy:

Eric: Darcy, how long have you been working for Farmer John’s Markets?

Darcy: Almost fi ve years this June. You just started two weeks ago . . . right?

Eric: Yes. Do you mind if I ask you a question?

Darcy: No, go ahead.

Eric: What I want to know is, have they always had this rule that if your cash register is short at the end of the day, you have to make up the shortage out of your own pocket?

Darcy: Yes, as long as I’ve been working here.

Eric: Well, it’s the pits. Last week I had to pay in almost $25.

Darcy: It’s not that big a deal. I just make sure that I’m not short at the end of the day.

Eric: How do you do that?

Darcy: I just shortchange a few customers early in the day. There are a few jerks that deserve it anyway. Most of the time, their attention is elsewhere and they don’t think to check their change.

Eric: What happens if you’re over at the end of the day?

Darcy: Nancy lets me keep it as long as it doesn’t get to be too large. I’ve not been short in over a year. I usually clear about $10 to $15 extra per day.

Discuss this case from the viewpoint of proper controls and professional behavior.

CP 7-5 Bank reconciliation and internal controlThe records of Diamondale Company indicate a May 31 cash balance of $10,550, which includes undeposited receipts for May 30 and 31. The cash balance on the bank state-ment as of May 31 is $8,575. This balance includes a note of $5,000 plus $200 interest collected by the bank but not recorded in the journal. Checks outstanding on May 31 were as follows: No. 670, $900; No. 679, $750; No. 690, $1,650; No. 1148, $225; No. 1149, $300; and No. 1151, $600.

On May 3, the cashier resigned, effective at the end of the month. Before leaving on May 31, the cashier prepared the following bank reconciliation:

Cash balance per books, May 31 . . . . . . . . . . . . . . . . . . . . . . . . . . . . . . . . . . . . $10,550Add outstanding checks:

No. 1148 . . . . . . . . . . . . . . . . . . . . . . . . . . . . . . . . . . . . . . . . . . . . . . . . . . . . . . . . $2251149 . . . . . . . . . . . . . . . . . . . . . . . . . . . . . . . . . . . . . . . . . . . . . . . . . . . . . . . . 3001151 . . . . . . . . . . . . . . . . . . . . . . . . . . . . . . . . . . . . . . . . . . . . . . . . . . . . . . . . 600 1,025

$11,575Less undeposited receipts . . . . . . . . . . . . . . . . . . . . . . . . . . . . . . . . . . . . . . . . . 3,000Cash balance per bank, May 31 . . . . . . . . . . . . . . . . . . . . . . . . . . . . . . . . . . . . . $ 8,575Deduct unrecorded note with interest . . . . . . . . . . . . . . . . . . . . . . . . . . . . . . 5,200True cash, May 31 . . . . . . . . . . . . . . . . . . . . . . . . . . . . . . . . . . . . . . . . . . . . . . . . . $ 3,375

Calculator Tape of Outstanding Checks:0*

225 +300 +600 +

1,025*

Subsequently, the owner of Diamondale Company discovered that the cashier had stolen an unknown amount of undeposited receipts, leaving only $1,000 to be deposited on May 31. The owner, a close family friend, has asked your help in determining the amount that the former cashier has stolen.

Page 20: Cases & Projects

Chapter 7 Sarbanes-Oxley, Internal Control, and Cash 357

1. Determine the amount the cashier stole from Diamondale Company. Show your com-putations in good form.

2. How did the cashier attempt to conceal the theft?

3. a. Identify two major weaknesses in internal controls, which allowed the cashier to steal the undeposited cash receipts.

b. Recommend improvements in internal controls, so that similar types of thefts of undeposited cash receipts can be prevented.

CP 7-6 Observe internal controls over cash

Group ProjectSelect a business in your community and observe its internal controls over cash receipts and cash payments. The business could be a bank or a bookstore, restaurant, department store, or other retailer. In groups of three or four, identify and discuss the similarities and differences in each business’s cash internal controls.

CP 7-7 Cash to monthly cash expenses ratioOccuLogix, Inc., is a health care company that specializes in developing diagnostic devices for eye disease. OccuLogix reported the following data (in thousands) for the years ending December 31, 2008, 2007, and 2006:

For Years Ending December 31

2008 2007 2006Cash and cash equivalents $ 2,565 $ 2,236 $ 5,741Net cash fl ows from operations (9,434) (17,217) (14,548)

1. Determine the monthly cash expenses for 2008, 2007, and 2006. Round to one decimal place.

2. Determine the ratio of cash to monthly cash expenses as of December 31, 2008, 2007, and 2006. Round to one decimal place.

3. Based on (1) and (2), comment on OccuLogix’s ratio of cash to monthly operating expenses for 2008, 2007, and 2006.

F•A•I

Page 21: Cases & Projects

Chapter 8 Receivables 399

June 23. Received $10,100 on note of March 25.

30. Received $25,125 on note of May 31.

July 1. Received a $28,000, 9%, 30-day note on account.

31. Received $28,210 on note of July 1.

Aug. 14. Received $36,630 on note of May 16.

InstructionsJournalize the entries to record the transactions.

PR 8-6B Sales and notes receivable transactionsThe following were selected from among the transactions completed during the current year by Indigo Co., an appliance wholesale company:

Jan. 13. Sold merchandise on account to Boylan Co., $32,000. The cost of merchandise sold was $19,200.

Mar. 10. Accepted a 60-day, 6% note for $32,000 from Boylan Co. on account.

May 9. Received from Boylan Co. the amount due on the note of March 10.

June 10. Sold merchandise on account to Holen for $18,000. The cost of merchandise sold was $10,000.

15. Loaned $24,000 cash to Angie Jones, receiving a 30-day, 7% note.

20. Received from Holen the amount due on the invoice of June 10, less 2% dis-count.

July 15. Received the interest due from Angie Jones and a new 60-day, 9% note as a renewal of the loan of June 15. (Record both the debit and the credit to the notes receivable account.)

Sept. 13. Received from Angie Jones the amount due on her note of July 15.

13. Sold merchandise on account to Aztec Co., $40,000. The cost of merchandise sold was $25,000.

Oct. 12. Accepted a 60-day, 6% note for $40,000 from Aztec Co. on account.

Dec. 11. Aztec Co. dishonored the note dated October 12.

26. Received from Aztec Co. the amount owed on the dishonored note, plus inter-est for 15 days at 12% computed on the maturity value of the note.

InstructionsJournalize the transactions.

OBJ. 6

Cases & Projects

CP 8-1 Ethics and professional conduct in businessStacey Ball, vice president of operations for Clinton County Bank, has instructed the bank’s computer programmer to use a 365-day year to compute interest on depository accounts (liabilities). Stacey also instructed the programmer to use a 360-day year to compute interest on loans (assets).

Discuss whether Stacey is behaving in a professional manner.

CP 8-2 Estimate uncollectible accountsFor several years, Dolphin Co.’s sales have been on a “cash only” basis. On January 1, 2009, however, Dolphin Co. began offering credit on terms of n/30. The amount of the adjusting entry to record the estimated uncollectible receivables at the end of each year

amazzaro
Text Box
Chapter 8
Page 22: Cases & Projects

400 Chapter 8 Receivables

has been ¼ of 1% of credit sales, which is the rate reported as the average for the in-dustry. Credit sales and the year-end credit balances in Allowance for Doubtful Accounts for the past four years are as follows:

Year Credit SalesAllowance for

Doubtful Accounts2009 $3,000,000 $ 3,2002010 3,150,000 5,5002011 3,400,000 8,0002012 3,800,000 10,300

Hugh Lopez, president of Dolphin Co., is concerned that the method used to account for and write off uncollectible receivables is unsatisfactory. He has asked for your advice in the analysis of past operations in this area and for recommendations for change.

1. Determine the amount of (a) the addition to Allowance for Doubtful Accounts and (b) the accounts written off for each of the four years.

2. a. Advise Hugh Lopez as to whether the estimate of ¼ of 1% of credit sales ap-pears reasonable.

b. Assume that after discussing (a) with Hugh Lopez, he asked you what action might be taken to determine what the balance of Allowance for Doubtful Accounts should be at December 31, 2012, and what possible changes, if any, you might recom-mend in accounting for uncollectible receivables. How would you respond?

CP 8-3 Accounts receivable turnover and days’ sales in receivablesBest Buy is a specialty retailer of consumer electronics, including personal computers, entertainment software, and appliances. Best Buy operates retail stores in addition to the Best Buy, Media Play, On Cue, and Magnolia Hi-Fi Web sites. For two recent years, Best Buy reported the following (in millions):

Year Ending

Feb. 28, 2009 Mar. 1, 2008Net sales $45,015 $40,023Accounts receivable at end of year 1,868 549

Assume that the accounts receivable (in millions) were $548 at the beginning of fiscal year 2008.

1. Compute the accounts receivable turnover for 2009 and 2008. Round to one decimal place.

2. Compute the days’ sales in receivables at the end of 2009 and 2008. Round to one decimal place.

3. What conclusions can be drawn from (1) and (2) regarding Best Buy’s effi-ciency in collecting receivables?

4. What assumption did we make about sales for the Best Buy ratio computa-tions that might distort the ratios and therefore cause the ratios not to be comparable for 2009 and 2008?

CP 8-4 Accounts receivable turnover and days’ sales in receivablesApple Inc. designs, manufactures, and markets personal computers and related personal computing and communicating solutions for sale primarily to education, creative, con-sumer, and business customers. Substantially all of the company’s net sales over the last five years are from sales of its Macs, iPods, iPads, and related software and peripherals. For two recent fiscal years, Apple reported the following (in millions):

Year Ending

Sept. 26, 2009 Sept. 27, 2008Net sales $36,537 $32,479Accounts receivable at end of year 3,361 2,422

F•A•I

F•A•I

Page 23: Cases & Projects

Chapter 8 Receivables 401

Assume that the accounts receivable (in millions) were $1,637 at the beginning of fiscal year 2008.

1. Compute the accounts receivable turnover for 2009 and 2008. Round to one decimal place.

2. Compute the days’ sales in receivables at the end of 2009 and 2008. Round to one decimal place.

3. What conclusions can be drawn from (1) and (2) regarding Apple’s efficiency in collecting receivables?

CP 8-5 Accounts receivable turnover and days’ sales in receivablesEarthLink, Inc., is a nationwide Internet Service Provider (ISP). EarthLink provides a variety of services to its customers, including narrowband access, broadband or high-speed ac-cess, and Web hosting services. For two recent years, EarthLink reported the following (in thousands):

Year Ending

Dec. 31, 2009 Dec. 31, 2008

Net sales $723,729 $955,577Accounts receivable at end of year 66,623 50,823

Assume that the accounts receivable (in thousands) were $41,483 at January 1, 2008.

1. Compute the accounts receivable turnover for 2009 and 2008. Round to one decimal place.

2. Compute the days’ sales in receivables at the end of 2009 and 2008. Round to one decimal place.

3. What conclusions can be drawn from (1) and (2) regarding EarthLink’s effi-ciency in collecting receivables?

4. Given the nature of EarthLink’s operations, do you believe EarthLink’s ac-counts receivable turnover ratio would be higher or lower than a typical manufacturing company, such as Boeing or Kellogg Company? Explain.

CP 8-6 Accounts receivable turnoverThe accounts receivable turnover ratio will vary across companies, depending on the na-ture of the company’s operations. For example, an accounts receivable turnover of 6 for an Internet Service Provider is unacceptable but might be excellent for a manufacturer of specialty milling equipment. A list of well-known companies follows.

Alcoa Inc. The Coca-Cola Company KrogerAutoZone, Inc. Delta Air Lines Procter & GambleBarnes & Noble, Inc. The Home Depot Wal-MartCaterpillar IBM Whirlpool Corporation

1. Categorize each of the preceding companies as to whether its turnover ratio is likely to be above or below 15.

2. Based on (1), identify a characteristic of companies with accounts receivable turnover ratios above 15.

F•A•I

F•A•I

Page 24: Cases & Projects

Chapter 9 Fixed Assets and Intangible Assets 445

2012

July 1. Purchased a new truck for $64,000, paying cash.

Oct. 7. Sold the truck purchased January 3, 2011, for $45,000. (Record depreciation for the year.)

Dec. 31. Recorded depreciation on the remaining truck. It has an estimated residual value of $17,500 and an estimated useful life of 10 years.

InstructionsJournalize the transactions and the adjusting entries.

PR 9-6B Amortization and depletion entriesData related to the acquisition of timber rights and intangible assets during the current year ended December 31 are as follows:

a. On December 31, the company determined that $1,800,000 of goodwill was impaired.

b. Governmental and legal costs of $900,000 were incurred on June 30 in obtaining a patent with an estimated economic life of 10 years. Amortization is to be for one-half year.

c. Timber rights on a tract of land were purchased for $1,560,000 on February 4. The stand of timber is estimated at 12,000,000 board feet. During the current year, 3,200,000 board feet of timber were cut and sold.

Instructions1. Determine the amount of the amortization, depletion, or impairment for the current

year for each of the foregoing items.

2. Journalize the adjusting entries to record the amortization, depletion, or impairment for each item.

OBJ. 4, 5

b. $45,000

Cases & Projects

CP 9-1 Ethics and professional conduct in businessRosa Salinas, CPA, is an assistant to the controller of Zebra Consulting Co. In her spare time, Rosa also prepares tax returns and performs general accounting services for clients. Frequently, Rosa performs these services after her normal working hours, using Zebra Consulting Co.’s computers and laser printers. Occasionally, Rosa’s clients will call her at the office during regular working hours.

Discuss whether Rosa is performing in a professional manner.

CP 9-2 Financial vs. tax depreciationThe following is an excerpt from a conversation between two employees of Omni Tech-nologies, Jay Bach and Cora Hardaway. Jay is the accounts payable clerk, and Cora is the cashier.

Jay: Cora, could I get your opinion on something?

Cora: Sure, Jay.

Jay: Do you know Jo, the fi xed assets clerk?

Cora: I know who she is, but I don’t know her real well. Why?

Jay: Well, I was talking to her at lunch last Monday about how she liked her job, etc. You know, the usual . . . and she mentioned something about having to keep two sets of books . . . one for taxes and one for the fi nancial state-ments. That can’t be good accounting, can it? What do you think?

Cora: Two sets of books? It doesn’t sound right.

Jay: It doesn’t seem right to me either. I was always taught that you had to use generally accepted accounting prin-ciples. How can there be two sets of books? What can be the diff erence between the two?

How would you respond to Jay and Cora if you were Jo?

amazzaro
Text Box
Chapter 9
Page 25: Cases & Projects

446 Chapter 9 Fixed Assets and Intangible Assets

CP 9-3 Effect of depreciation on net incomeAtlas Construction Co. specializes in building replicas of historic houses. Paul Raines, president of Atlas Construction, is considering the purchase of various items of equipment on July 1, 2010, for $500,000. The equipment would have a useful life of five years and no residual value. In the past, all equipment has been leased. For tax purposes, Paul is considering depreciating the equipment by the straight-line method. He discussed the matter with his CPA and learned that, although the straight-line method could be elected, it was to his advantage to use the Modified Accelerated Cost Recovery System (MACRS) for tax purposes. He asked for your advice as to which method to use for tax purposes.

1. Compute depreciation for each of the years (2010, 2011, 2012, 2013, 2014, and 2015) of useful life by (a) the straight-line method and (b) MACRS. In using the straight-line method, one-half year’s depreciation should be computed for 2010 and 2015. Use the MACRS rates presented on page 415.

2. Assuming that income before depreciation and income tax is estimated to be $900,000 uniformly per year and that the income tax rate is 40%, compute the net income for each of the years 2010, 2011, 2012, 2013, 2014, and 2015 if (a) the straight-line method is used and (b) MACRS is used.

3. What factors would you present for Paul’s consideration in the selection of a depreciation method?

CP 9-4 Applying for patents, copyrights, and trademarks

Group ProjectGo to the Internet and review the procedures for applying for a patent, a copyright, and a trademark. You may find information available on Wikipedia (Wikipedia.org) useful for this purpose. Prepare a brief written summary of these procedures.

CP 9-5 Fixed asset turnover: Three industriesThe following table shows the revenues and average net fixed assets for a recent fiscal year for three different companies from three different industries: retailing, manufactur-ing, and communications.

Revenues (in millions)

Average Net Fixed Assets (in millions)

Wal-Mart $405,607 $96,335Occidental Petroleum Corporation 15,403 32,856Comcast Corporation 34,256 24,034

a. For each company, determine the fixed asset turnover ratio. Round to two decimal places.

b. Explain Wal-Mart’s ratio relative to the other two companies.

Internet Project

F•A•I

Page 26: Cases & Projects

494 Chapter 10 Current Liabilities and Payroll

CP 10-1 Ethics and professional conduct in businessLisa Deuel is a certified public accountant (CPA) and staff accountant for Bratz and Bratz, a local CPA firm. It had been the policy of the firm to provide a holiday bonus equal to two weeks’ salary to all employees. The firm’s new management team announced on November 15 that a bonus equal to only one week’s salary would be made available to employees this year. Lisa thought that this policy was unfair because she and her co-workers planned on the full two-week bonus. The two-week bonus had been given for 10 straight years, so it seemed as though the firm had breached an implied commitment. Thus, Lisa decided that she would make up the lost bonus week by working an extra six hours of overtime per week over the next five weeks until the end of the year. Bratz and Bratz’s policy is to pay overtime at 150% of straight time.

Lisa’s supervisor was surprised to see overtime being reported, since there is gener-ally very little additional or unusual client service demands at the end of the calendar year. However, the overtime was not questioned, since firm employees are on the “honor system” in reporting their overtime.

Discuss whether the firm is acting in an ethical manner by changing the bonus. Is Lisa behaving in an ethical manner?

CP 10-2 Recognizing pension expenseThe annual examination of Wave Company’s financial statements by its external public accounting firm (auditors) is nearing completion. The following conversation took place between the controller of Wave Company (Tommy) and the audit manager from the public accounting firm (Jaclyn).

Jaclyn: You know, Tommy, we are about to wrap up our audit for this fi scal year. Yet, there is one item still to be resolved.

Tommy: What’s that?

Jaclyn: Well, as you know, at the beginning of the year, Wave began a defi ned benefi t pension plan. This plan promises your employees an annual payment when they retire, using a formula based on their salaries at retirement and their years of service. I believe that a pension expense should be recognized this year, equal to the amount of pension earned by your employees.

Tommy: Wait a minute. I think you have it all wrong. The company doesn’t have a pension expense until it actually pays the pension in cash when the employee retires. After all, some of these employees may not reach retirement, and if they don’t, the company doesn’t owe them anything.

Jaclyn: You’re not really seeing this the right way. The pension is earned by your employees during their working years. You actually make the payment much later—when they retire. It’s like one long accrual—much like incurring wages in one period and paying them in the next. Thus, I think that you should recognize the expense in the period the pension is earned by the employees.

Tommy: Let me see if I’ve got this straight. I should recognize an expense this period for something that may or may not be paid to the employees in 20 or 30 years, when they fi nally retire. How am I supposed to determine what the expense is for the current year? The amount of the fi nal retirement depends on many uncertainties: salary levels, employee longevity, mortality rates, and interest earned on investments to fund the pension. I don’t think that an amount can be determined, even if I accepted your arguments.

Evaluate Jaclyn’s position. Is she right or is Tommy correct?

Cases & Projects

amazzaro
Text Box
Chapter 10
Page 27: Cases & Projects

Chapter 10 Current Liabilities and Payroll 495

CP 10-3 Ethics and professional conduct in businessGloria Seuss was discussing summer employment with Ella Kitt, president of Hotel Cali-fornia Construction Service:

Ella: I’m glad that you’re thinking about joining us for the summer. We could certainly use the help.

Gloria: Sounds good. I enjoy outdoor work, and I could use the money to help with next year’s school expenses.

Ella: I’ve got a plan that can help you out on that. As you know, I’ll pay you $14 per hour, but in addition, I’d like to pay you with cash. Since you’re only working for the summer, it really doesn’t make sense for me to go to the trouble of formally putting you on our payroll system. In fact, I do some jobs for my clients on a strictly cash basis, so it would be easy to just pay you that way.

Gloria: Well, that’s a bit unusual, but I guess money is money.

Ella: Yeah, not only that, it’s tax-free!

Gloria: What do you mean?

Ella: Didn’t you know? Any money that you receive in cash is not reported to the IRS on a W-2 form; therefore, the IRS doesn’t know about the income—hence, it’s the same as tax-free earnings.

a. Why does Ella Kitt want to conduct business transactions using cash (not check or credit card)?

b. How should Gloria respond to Ella’s suggestion?

CP 10-4 Payroll forms

Group ProjectPayroll accounting involves the use of government-supplied forms to account for payroll taxes. Three common forms are the W-2, Form 940, and Form 941. Form a team with three of your classmates and retrieve copies of each of these forms. They may be obtained from a local IRS office, a library, or downloaded from the Internet at http://www.irs.gov (go to forms and publications).

Briefly describe the purpose of each of the three forms.

CP 10-5 Contingent liabilitiesAltria Group, Inc., has over 12 pages dedicated to describing contingent liabilities in the notes to recent financial statements. These pages include extensive descriptions of mul-tiple contingent liabilities. Use the Internet to research Altria Group, Inc., at http://www.altria.com.

a. What are the major business units of Altria Group?

b. Based on your understanding of this company, why would Altria Group require 11 pages of contingency disclosure?

Internet Project

Internet Project

Page 28: Cases & Projects

Chapter 11 Corporations: Organization, Stock Transactions, and Dividends 535

Cases & Projects

CP 11-1 Board of directors’ actionsBernie Ebbers, the CEO of WorldCom, a major telecommunications company, was having personal financial troubles. Ebbers pledged a large stake of his WorldCom stock as security for some personal loans. As the price of WorldCom stock sank, Ebbers’ bankers threatened to sell his stock in order to protect their loans. To avoid having his stock sold, Ebbers asked the board of directors of WorldCom to loan him nearly $400 million of corporate assets at 2.5% interest to pay off his bankers. The board agreed to lend him the money.

Comment on the decision of the board of directors in this situation.

CP 11-2 Ethics and professional conduct in businessHazel Holden and Cedric Dalton are organizing Calgary Metals Unlimited Inc. to undertake a high-risk gold-mining venture in Canada. Hazel and Cedric tentatively plan to request authorization for 100,000,000 shares of common stock to be sold to the general public. Hazel and Cedric have decided to establish par of $0.02 per share in order to appeal to a wide variety of potential investors. Hazel and Cedric feel that investors would be more willing to invest in the company if they received a large quantity of shares for what might appear to be a “bargain” price.

Discuss whether Hazel and Cedric are behaving in a professional manner.

CP 11-3 Issuing stockBio Engineering Inc. began operations on January 2, 2012, with the issuance of 250,000 shares of $80 par common stock. The sole stockholders of Bio Engineering Inc. are Jean Cushing and Dr. Louis Fong, who organized Bio Engineering Inc. with the objective of developing a new flu vaccine. Dr. Fong claims that the flu vaccine, which is nearing the final development stage, will protect individuals against 95% of the flu types that have been medically identified. To complete the project, Bio Engineering Inc. needs $40,000,000 of additional funds. The local banks have been unwilling to loan the funds because of the lack of sufficient collateral and the riskiness of the business.

The following is a conversation between Jean Cushing, the chief executive officer of Bio Engineering Inc., and Louis Fong, the leading researcher.

Jean: What are we going to do? The banks won’t loan us any more money, and we’ve got to have $40 million to complete the project. We are so close! It would be a disaster to quit now. The only thing I can think of is to issue additional stock. Do you have any suggestions?

Louis: I guess you’re right. But if the banks won’t loan us any more money, how do you think we can find any investors to buy stock?

Jean: I’ve been thinking about that. What if we promise the investors that we will pay them 5% of net sales until they have received an amount equal to what they paid for the stock?

Louis: What happens when we pay back the $40 million? Do the investors get to keep the stock? If they do, it’ll dilute our ownership.

Jean: How about, if after we pay back the $40 million, we make them turn in their stock for $120 per share? That’s one and one-half times what they paid for it, plus they would have already gotten all their money back. That’s a $120 profit per share for the investors.

Louis: It could work. We get our money, but don’t have to pay any interest, dividends, or the $80 per share until we start generating net sales. At the same time, the investors could get their money back plus $120 per share profit.

Jean: We’ll need current financial statements for the new investors. I’ll get our accountant working on them and contact our attorney to draw up a legally binding contract for the new investors. Yes, this could work.

In late 2012, the attorney and the various regulatory authorities approved the new stock offering, and 500,000 shares of common stock were privately sold to new investors at the stock’s par of $80.

In preparing financial statements for 2012, Jean Cushing and Todd Nash, the controller for Bio Engineering Inc., have the following conversation:

amazzaro
Text Box
Chapter 11
Page 29: Cases & Projects

536 Chapter 11 Corporations: Organization, Stock Transactions, and Dividends

Todd: Jean, I’ve got a problem.

Jean: What’s that, Todd?

Todd: Issuing common stock to raise that additional $40 million was a great idea. But . . .

Jean: But what?

Todd: I’ve got to prepare the 2012 annual financial statements, and I am not sure how to classify the common stock.

Jean: What do you mean? It’s common stock.

Todd: I’m not so sure. I called the auditor and explained how we are contractually obligated to pay the new stock-holders 5% of net sales until $80 per share is paid. Then, we may be obligated to pay them $120 per share.

Jean: So . . .

Todd: So the auditor thinks that we should classify the additional issuance of $40 million as debt, not stock! And, if we put the $40 million on the balance sheet as debt, we will violate our other loan agreements with the banks. And, if these agreements are violated, the banks may call in all our debt immediately. If they do that, we are in deep trouble. We’ll probably have to file for bankruptcy. We just don’t have the cash to pay off the banks.

1. Discuss the arguments for and against classifying the issuance of the $40 million of stock as debt.

2. What do you think might be a practical solution to this classification problem?

CP 11-4 Interpret stock exchange listingThe following stock exchange data for General Electric (GE) were taken from the Yahoo! Finance Web site on April 29, 2010:

Gen Electric Co (NYSE: GE)Last Trade: 19.49 Prev. Clos: 18.95Trade Time: 4:00 PM EST 1y Target Est: 22.09Change: 0.54 Day’s Range: 19.03–19.49

(2.85%) 52wk Range: 10.50–19.69Volume: 70,066,312Div & Yield: 0.40 (2.40%)

a. If you owned 500 shares of GE, what amount would you receive as a quarterly dividend?

b. Compute the percentage increase in price from the Previous Close to the Last Trade. Round to two decimal places.

c. What is GE’s percentage change in market price from the 52-week low to the Previous Close on April 28, 2010? Round to one decimal place.

d. If you bought 500 shares of GE at the Last Trade price on April 29, 2010, how much would it cost, and who gets the money?

CP 11-5 DividendsCikan Designs Inc. has paid quarterly cash dividends since 2001. These dividends have steadily increased from $0.02 per share to the latest dividend declaration of $0.40 per share. The board of directors would like to continue this trend and is hesitant to suspend or decrease the amount of quarterly dividends. Unfortunately, sales dropped sharply in the fourth quarter of 2012 because of worsening economic conditions and increased competition. As a result, the board is uncertain as to whether it should declare a dividend for the last quarter of 2012.

On November 1, 2012, Cikan Designs Inc. borrowed $3,600,000 from Metro National Bank to use in modernizing its retail stores and to expand its product line in reaction to its competition. The terms of the 10-year, 6% loan require Cikan Designs Inc. to:

a. Pay monthly interest on the last day of the month.

b. Pay $360,000 of the principal each November 1, beginning in 2013.

c. Maintain a current ratio (current assets/current liabilities) of 2.

d. Maintain a minimum balance (a compensating balance) of $100,000 in its Metro National Bank account.

F•A•I

Page 30: Cases & Projects

Chapter 11 Corporations: Organization, Stock Transactions, and Dividends 537

On December 31, 2012, $900,000 of the $3,600,000 loan had been disbursed in modernization of the retail stores and in expansion of the product line. Cikan Designs Inc.’s balance sheet as of December 31, 2012, is shown below.

Cikan Designs Inc.Balance Sheet

December 31, 2012Assets

Current assets: Cash . . . . . . . . . . . . . . . . . . . . . . . . . . . . . . . . . . . . . $ 250,000 Marketable securities . . . . . . . . . . . . . . . . . . . . . 2,700,000 Accounts receivable . . . . . . . . . . . . . . . . . . . . . . . $ 700,000 Less allowance for doubtful accounts . . . . 50,000 650,000 Merchandise inventory . . . . . . . . . . . . . . . . . . . . 2,680,000 Prepaid expenses . . . . . . . . . . . . . . . . . . . . . . . . . 20,000 Total current assets . . . . . . . . . . . . . . . . . . . . . $ 6,300,000Property, plant, and equipment: Land . . . . . . . . . . . . . . . . . . . . . . . . . . . . . . . . . . . . . $ 500,000 Buildings . . . . . . . . . . . . . . . . . . . . . . . . . . . . . . . . . $4,750,000 Less accumulated depreciation. . . . . . . . . . . 1,140,000 3,610,000 Equipment . . . . . . . . . . . . . . . . . . . . . . . . . . . . . . . $2,320,000 Less accumulated depreciation . . . . . . . . . . 730,000 1,590,000 Total property, plant, and equipment . . . . . 5,700,000Total assets. . . . . . . . . . . . . . . . . . . . . . . . . . . . . . . . . . . . . $12,000,000

liabilitiesCurrent liabilities: Accounts payable . . . . . . . . . . . . . . . . . . . . . . . . . $1,430,000 Notes payable (Metro National Bank). . . . . . . . 360,000 Salaries payable . . . . . . . . . . . . . . . . . . . . . . . . . . . 10,000 Total current liabilities . . . . . . . . . . . . . . . . . . . $1,800,000Long-term liabilities: Notes payable (Metro National Bank). . . . . . . . 3,240,000Total liabilities . . . . . . . . . . . . . . . . . . . . . . . . . . . . . . $ 5,040,000

Stockholders’ EquityPaid-in capital: Common stock, $25 par (200,000 shares authorized, 180,000 shares issued) . . . . . . . . $4,500,000 Excess of issue price over par . . . . . . . . . . . . . . . 270,000 Total paid-in capital . . . . . . . . . . . . . . . . . . . . . $4,770,000Retained earnings . . . . . . . . . . . . . . . . . . . . . . . . . . . 2,190,000Total stockholders’ equity . . . . . . . . . . . . . . . . . . . . 6,960,000Total liabilities and stockholders’ equity . . . . . . . $12,000,000

The board of directors is scheduled to meet January 8, 2013, to discuss the results of operations for 2012 and to consider the declaration of dividends for the fourth quarter of 2012. The chairman of the board has asked for your advice on the declaration of dividends.

1. What factors should the board consider in deciding whether to declare a cash dividend?

2. The board is considering the declaration of a stock dividend instead of a cash dividend. Discuss the issuance of a stock dividend from the point of view of (a) a stockholder and (b) the board of directors.

CP 11-6 Profiling a corporation

Group ProjectSelect a public corporation you are familiar with or which interests you. Using the Inter-net, your school library, and other sources, develop a short (1 to 2 pages) profile of the corporation. Include in your profile the following information:

Internet Project

Page 31: Cases & Projects

538 Chapter 11 Corporations: Organization, Stock Transactions, and Dividends

1. Name of the corporation.

2. State of incorporation.

3. Nature of its operations.

4. Total assets for the most recent balance sheet.

5. Total revenues for the most recent income statement.

6. Net income for the most recent income statement.

7. Classes of stock outstanding.

8. Market price of the stock outstanding.

9. High and low price of the stock for the past year.

10. Dividends paid for each share of stock during the past year.In groups of three or four, discuss each corporate profile. Select one of the corpora-

tions, assuming that your group has $100,000 to invest in its stock. Summarize why your group selected the corporation it did and how financial accounting information may have affected your decision. Keep track of the performance of your corporation’s stock for the remainder of the term.

Note: Most major corporations maintain “home pages” on the Internet. This home page provides a variety of information on the corporation and often includes the corporation’s fi-nancial statements. In addition, the New York Stock Exchange Web site (http://www.nyse.com) includes links to the home pages of many listed companies. Financial statements can also be accessed using EDGAR, the electronic archives of financial statements filed with the Securities and Exchange Commission (SEC).

SEC documents can also be retrieved using the EdgarScan™ service at http://www.sec.gov/edgar/searchedgar/companysearch.html. To obtain annual report information, key in a company name in the appropriate space. Edgar will list the reports available to you for the company you’ve selected. Select the most recent annual report filing, identified as a 10-K or 10-K405.

Page 32: Cases & Projects

578 Chapter 12 Long-Term Liabilities: Bonds and Notes

Cases & Projects

CP 12-1 General Electric bond issuance

General Electric Capital, a division of General Electric, uses long-term debt extensively. In a recent year, GE Capital issued $11 billion in long-term debt to investors, then within days filed legal documents to prepare for another $50 billion long-term debt issue. As a result of the $50 billion filing, the price of the initial $11 billion offering declined (due to higher risk of more debt).

Bill Gross, a manager of a bond investment fund, “denounced a ‘lack in candor’ related to GE’s recent debt deal. ‘It was the most recent and most egregious example of how bondholders are mistreated.’ Gross argued that GE was not forth-right when GE Capital recently issued $11 billion in bonds, one of the largest issues ever from a U.S. corporation. What bothered Gross is that three days after the issue the company announced its intention to sell as much as $50 billion in additional debt, warrants, preferred stock, guarantees, letters of credit and promissory notes at some future date.”

In your opinion, did GE Capital act unethically by selling $11 billion of long-term debt without telling those investors that a few days later it would be filing documents to prepare for another $50 billion debt offering?Source: Jennifer Ablan, “Gross Shakes the Bond Market; GE Calms It, a Bit,” Barron’s, March 25, 2002.

CP 12-2 Ethics and professional conduct in business Juicy Energy Industries develops and produces biomass, an alternative energy source. The company has an outstanding $10,000,000, 30-year, 10% bond issue dated July 1, 2007. The bond issue is due June 30, 2037. Some bond indentures require the corporation issuing the bonds to transfer cash to a special cash fund, called a sinking fund, over the life of the bond. Such funds help assure investors that there will be adequate cash to pay the bonds at their maturity date.

The bond indenture requires a bond sinking fund, which has a balance of $1,200,000 as of July 1, 2012. The company is currently experiencing a shortage of funds due to a recent acquisition. Dillip Fogel, the company’s treasurer, is considering using the funds from the bond sinking fund to cover payroll and other bills that are coming due at the end of the month. Dillip’s brother-in-law, a trustee in a sinking fund, has indicated will-ingness to allow Dillip to use the funds from the sinking fund to temporarily meet the company’s cash needs.

Discuss whether Dillip’s proposal is appropriate.

CP 12-3 Present valuesBailey Mills recently won the jackpot in the New Jersey lottery while he was visiting his parents. When he arrived at the lottery office to collect his winnings, he was offered the following three payout options:

a. Receive $40,000,000 in cash today.

b. Receive $10,000,000 today and $3,600,000 per year for 10 years, with the first payment being received one year from today.

c. Receive $5,000,000 per year for 20 years, with the first payment being received one year from today.

Assuming that the effective rate of interest is 10%, which payout option should Bailey select? Explain your answer and provide any necessary supporting calculations.

CP 12-4 Preferred stock vs. bondsAccusport Inc. has decided to expand its operations to owning and operating golf courses. The following is an excerpt from a conversation between the chief executive officer, Tucker Thorup, and the vice president of finance, Don Clark.

amazzaro
Text Box
Chapter 12
Page 33: Cases & Projects

Chapter 12 Long-Term Liabilities: Bonds and Notes 579

Tucker: Don, have you given any thought to how we’re going to finance the acquisition of Knotty Pines Golf Course?

Don: Well, the two basic options, as I see it, are to issue either preferred stock or bonds. The equity market is a little depressed right now. The rumor is that the Federal Reserve Bank’s going to increase the interest rates either this month or next.

Tucker: Yes, I’ve heard the rumor. The problem is that we can’t wait around to see what’s going to happen. We’ll have to move on this next week if we want any chance to complete the acquisition of Knotty Pines Golf Course.

Don: Well, the bond market is strong right now. Maybe we should issue debt this time around.

Tucker: That’s what I would have guessed as well. Knotty Pines Golf Course’s financial statements look pretty good, except for the volatility of its income and cash flows. But that’s characteristic of the industry.

Discuss the advantages and disadvantages of issuing preferred stock versus bonds.

CP 12-5 Financing business expansionYou hold a 25% common stock interest in the family-owned business, a construction equipment company. Your sister, who is the manager, has proposed an expansion of plant facilities at an expected cost of $10,000,000. Two alternative plans have been sug-gested as methods of financing the expansion. Each plan is briefly described as follows:

Plan 1. Issue $10,000,000 of 20-year, 10% notes at face amount.

Plan 2. Issue an additional 200,000 shares of $10 par common stock at $20 per share, and $6,000,000 of 20-year, 10% notes at face amount.

The balance sheet as of the end of the previous fiscal year is as follows:

Thacker, Inc. Balance Sheet

December 31, 2012Assets

Current assets . . . . . . . . . . . . . . . . . . . . . . . . . . . . . . . . . . . . . . . . . . . . . . . . . . . . . . . . . . . $ 6,000,000Property, plant, and equipment . . . . . . . . . . . . . . . . . . . . . . . . . . . . . . . . . . . . . . . . . . . 9,000,000

Total assets . . . . . . . . . . . . . . . . . . . . . . . . . . . . . . . . . . . . . . . . . . . . . . . . . . . . . . . . . . . $15,000,000liabilities and Stockholders’ Equity

Liabilities . . . . . . . . . . . . . . . . . . . . . . . . . . . . . . . . . . . . . . . . . . . . . . . . . . . . . . . . . . . . . . . . $ 4,500,000Common stock, $10 . . . . . . . . . . . . . . . . . . . . . . . . . . . . . . . . . . . . . . . . . . . . . . . . . . . . . . 1,600,000Paid-in capital in excess of par . . . . . . . . . . . . . . . . . . . . . . . . . . . . . . . . . . . . . . . . . . . . 200,000Retained earnings . . . . . . . . . . . . . . . . . . . . . . . . . . . . . . . . . . . . . . . . . . . . . . . . . . . . . . . . 8,700,000

Total liabilities and stockholders’ equity . . . . . . . . . . . . . . . . . . . . . . . . . . . . . . . . $15,000,000

Net income has remained relatively constant over the past several years. The expansion program is expected to increase yearly income before bond interest and income tax from $800,000 in the previous year to $1,500,000 for this year. Your sister has asked you, as the company treasurer, to prepare an analysis of each financing plan.

1. Prepare a table indicating the expected earnings per share on the common stock under each plan. Assume an income tax rate of 40%. Round to the nearest cent.

2. a. Discuss the factors that should be considered in evaluating the two plans.

b. Which plan offers the greater benefit to the present stockholders? Give reasons for your opinion.

CP 12-6 number of times interest charges are earned The following financial data were taken from the financial statements of Williams-Sonoma, Inc.

Fiscal Year2009 2008 2007

Interest expense . . . . . . . . . . . . . . . . . . . . . . . . . . . . . . . . . . . . . . . . . . . . $ 1,480 $ 2,099 $ 2,125Earnings before taxes . . . . . . . . . . . . . . . . . . . . . . . . . . . . . . . . . . . . . . . 41,953 316,340 337,186

1. What is the number of times interest charges are earned for Williams-Sonoma in 2009, 2008, and 2007? (Round your answers to one decimal place.)

2. Evaluate this ratio for Williams-Sonoma.

Page 34: Cases & Projects

626 Chapter 13 Investments and Fair Value Accounting

Cases & Projects

CP 13-1 Benefits of fair valueOn August 16, 1997, Parson Corp. purchased 20 acres of land for $500,000. The land has been held for a future plant site until the current date, December 31, 2012. On December 5, 2012, Overland, Inc., purchased 20 acres of land for $3,000,000 to be used for a dis-tribution center. The Overland land is located next to the Parson Corp. land. Thus, both Parson Corp. and Overland, Inc., own nearly identical pieces of land.

1. What are the valuations of land on the balance sheets of Parson Corp. and Overland, Inc., using generally accepted accounting principles?

2. How might fair value accounting aid comparability when evaluating these two com-panies?

CP 13-2 International fair value accountingInternational Accounting Standard No. 16 provides companies the option of valuing prop-erty, plant, and equipment at either historical cost or fair value. If fair value is selected, then the property, plant, and equipment must be revalued periodically to fair value. Under fair value, if there is an increase in the value of the property, plant, and equipment over the reporting period, then the increase is credited to stockholders’ equity. However, if there is a decrease in fair value, then the decrease is reported as an expense for the period.

1. Why do International Accounting Standards influence U.S. GAAP?

2. What would be some of the disadvantages of using fair value accounting for property, plant, and equipment?

3. How is the international accounting treatment for changes in fair value for property, plant, and equipment similar to investments?

CP 13-3 Ethics and fair value measurementFinancial assets include stocks and bonds. These are fairly simple securities that can often be valued using quoted market prices. However, Wall Street has created many complex and exotic securities that do not have quoted market prices. These complex securities must still be valued on the balance sheet at fair value. Generally accepted accounting principles require that the reporting entity use assumptions in valuing investments when market prices or critical valuation inputs are unobservable.

What are the ethical considerations in making subjective valuations of complex and exotic investments?

CP 13-4 Warren Buffett and “look-through” earningsBerkshire Hathaway, the investment holding company of Warren Buffett, reports its “less than 20% ownership” investments according to generally accepted accounting principles. However, it also provides additional disclosures that it terms “look-through” earnings.

Warren Buffett states,Many of these companies (in the less than 20% owned category) pay out relatively small proportions of their earnings in dividends. This means that only a small proportion of their earning power is recorded in our own current operating earn-ings. But, while our reported operating earnings refl ect only the dividends received from such companies, our economic well-being is determined by their earnings, not their dividends.

The value to Berkshire Hathaway of retained earnings (of our investees) is not determined by whether we own 100%, 50%, 20%, or 1% of the businesses in which they reside.... Our perspective on such “forgotten-but-not-gone” earnings is simple: the way they are accounted for is of no importance, but their ownership and subsequent utilization is all- important. We care not whether the auditors hear a tree fall in the forest; we do care who owns the tree and what’s next done with it.

amazzaro
Text Box
Chapter 13
Page 35: Cases & Projects

Chapter 13 Investments and Fair Value Accounting 627

I believe the best way to think about our earnings is in terms of “look-through” results, calculated as follows: Take $250 million, which is roughly our share of the operating earnings retained by our investees (<20% ownership hold-ings); subtract... incremental taxes we would have owed had that $250 million been paid to us in dividends; then add the remainder, $220 million, to our reported earnings of $371 million. Thus our, “look-through” earnings were about $590 million.Source: Warren Buffett, The Essays of Warren Buffett: Lessons for Corporate America, edited by Lawrence A. Cunningham, pp. 180–183 (excerpted).

1. What are “look-through” earnings?

2. Why does Warren Buffett favor “look-through” earnings?

CP 13-5 Reporting investments

Group ProjectIn groups of three or four, find the latest annual report for Microsoft Corporation. The annual report can be found on the company’s Web site at http://www.microsoft.com/msft/default.mspx.

The notes to the financial statements include details of Microsoft’s investments. Find the notes that provide details of its investments (Note 4) and the income from its invest-ments (Note 3).

From these disclosures, answer the following questions:

1. What is the total cost of investments?

2. What is the fair value (recorded value) of investments?

3. What is the total unrealized gain from investments?

4. What is the total unrealized loss from investments?

5. What percent of total investments (at fair value) are:

a. Cash and equivalents

b. Short-term investments

c. Equity and other investments (long term)

6. What was the total combined dividend and interest revenue?

7. What was the recognized net gain or loss from sale of investments?

Internet Project

Page 36: Cases & Projects

Cases & Projects

Statement of cash flows—direct method applied toThe comparative balance sheet of Juras Equipment Co. for Dec. 31, 2013 and 2012, is:

Dec. 31, 2013 Dec. 31, 2012

Assets

Cash . . . . . . . . . . . . . . . . . . . . . . . . . . . . . . . . . . . . . . . . . . . . . . . . . . . . . $ 99,840 $ 67,680Accounts receivable (net) . . . . . . . . . . . . . . . . . . . . . . . . . . . . . . . . . . 292,560 265,680Inventories . . . . . . . . . . . . . . . . . . . . . . . . . . . . . . . . . . . . . . . . . . . . . . . 421,440 409,200Investments . . . . . . . . . . . . . . . . . . . . . . . . . . . . . . . . . . . . . . . . . . . . . . 0 144,000Land . . . . . . . . . . . . . . . . . . . . . . . . . . . . . . . . . . . . . . . . . . . . . . . . . . . . . 417,600 0Equipment. . . . . . . . . . . . . . . . . . . . . . . . . . . . . . . . . . . . . . . . . . . . . . . . 619,200 505,440Accumulated depreciation . . . . . . . . . . . . . . . . . . . . . . . . . . . . . . . . . (139,920) (119,040)

$1,710,720 $1,272,960Liabilities and Stockholders’ Equity

Accounts payable (merchandise creditors) . . . . . . . . . . . . . . . . . . $ 290,400 $ 274,080Accrued expenses payable (operating expenses) . . . . . . . . . . . . 43,200 37,920Dividends payable. . . . . . . . . . . . . . . . . . . . . . . . . . . . . . . . . . . . . . . . . 36,000 28,800Common stock, $1 par . . . . . . . . . . . . . . . . . . . . . . . . . . . . . . . . . . . . . 162,000 144,000Paid-in capital in excess of par—common stock . . . . . . . . . . . . . 594,000 288,000Retained earnings . . . . . . . . . . . . . . . . . . . . . . . . . . . . . . . . . . . . . . . . . 585,120 500,160

$1,710,720 $1,272,960

The income statement for the year ended December 31, 2013, is as follows:

Sales . . . . . . . . . . . . . . . . . . . . . . . . . . . . . . . . . . . . . . . . . . . . . . . . . . . . . $3,246,048Cost of merchandise sold . . . . . . . . . . . . . . . . . . . . . . . . . . . . . . . . . . 1,997,568Gross profit . . . . . . . . . . . . . . . . . . . . . . . . . . . . . . . . . . . . . . . . . . . . . . . $1,248,480Operating expenses:

Depreciation expense . . . . . . . . . . . . . . . . . . . . . . . . . . . . . . . . . . $ 20,880Other operating expenses . . . . . . . . . . . . . . . . . . . . . . . . . . . . . . 831,600

Total operating expenses . . . . . . . . . . . . . . . . . . . . . . . . . . . . 852,480Operating income . . . . . . . . . . . . . . . . . . . . . . . . . . . . . . . . . . . . . . . . . $ 396,000Other expenses:

Loss on sale of investments . . . . . . . . . . . . . . . . . . . . . . . . . . . . . (14,400)Income before income tax . . . . . . . . . . . . . . . . . . . . . . . . . . . . . . . . . $ 381,600Income tax expense . . . . . . . . . . . . . . . . . . . . . . . . . . . . . . . . . . . . . . . 152,640Net income . . . . . . . . . . . . . . . . . . . . . . . . . . . . . . . . . . . . . . . . . . . . . . . $ 228,960

The following additional information was taken from the records:

a. Equipment and land were acquired for cash.

b. There were no disposals of equipment during the year.

c. The investments were sold for $129,600 cash.

d. The common stock was issued for cash.

e. There was a $144,000 debit to Retained Earnings for cash dividends declared.

Instructions Prepare a statement of cash flows, using the direct method of presenting cash flows from operating activities.

ObJ. 3

Net cash flow from operating activities, $246,720

Ethics and professional conduct in businessChris Ruth, president of Fairazon Industries Inc., believes that reporting operating cash flow per share on the income statement would be a useful addition to the company’s just completed financial statements. The following discussion took place between Chris Ruth and Fairazon controller, Phil Tungsten, in January, after the close of the fiscal year.

Chris: I’ve been reviewing our financial statements for the last year. I am disappointed that our net income per share has dropped by 10% from last year. This won’t look good to our shareholders. Is there anything we can do about this?

692 Chapter 14 Statement of Cash Flows

PR 14-5B

CP 14-1

PR 14-1B

amazzaro
Text Box
Chapter 14
Page 37: Cases & Projects

Phil: What do you mean? The past is the past, and the numbers are in. There isn’t much that can be done about it. Our financial statements were prepared according to generally accepted accounting principles, and I don’t see much leeway for significant change at this point.

Chris: No, no. I’m not suggesting that we “cook the books.” But look at the cash flow from operating activities on the statement of cash flows. The cash flow from operating activities has increased by 20%. This is very good news—and, I might add, useful information. The higher cash flow from operating activities will give our creditors comfort.

Phil: Well, the cash flow from operating activities is on the statement of cash flows, so I guess users will be able to see the improved cash flow figures there.

Chris: This is true, but somehow I feel that this information should be given a much higher profile. I don’t like this information being “buried” in the statement of cash flows. You know as well as I do that many users will focus on the income statement. Therefore, I think we ought to include an operating cash flow per share number on the face of the income statement—someplace under the earnings per share number. In this way, users will get the complete picture of our operating performance. Yes, our earnings per share dropped this year, but our cash flow from operating activities improved! And all the information is in one place where users can see and compare the figures. What do you think?

Phil: I’ve never really thought about it like that before. I guess we could put the operating cash flow per share on the income statement, under the earnings per share. Users would really benefit from this disclosure. Thanks for the idea—I’ll start working on it.

Chris: Glad to be of service.

How would you interpret this situation? Is Phil behaving in an ethical and pro-fessional manner?

Using the statement of cash flowsYou are considering an investment in a new start-up company, Over Armour Inc., an Internet service provider. A review of the company’s financial statements reveals a negative retained earnings. In addition, it appears as though the company has been running a negative cash flow from operating activities since the company’s inception.

How is the company staying in business under these circumstances? Could this be a good investment?

Analysis of cash flow from operationsThe Retailing Division of Argon Clothing Inc. provided the following information on its cash flow from operations:

Net income $ 675,000Increase in accounts receivable (810,000)Increase in inventory (900,000)Decrease in accounts payable (135,000)Depreciation 150,000Cash flow from operating activities $(1,020,000)

The manager of the Retailing Division provided the accompanying memo with this report:

From: Senior Vice President, Retailing Division

I am pleased to report that we had earnings of $675,000 over the last period. This resulted in a return on invested capital of 10%, which is near our targets for this division. I have been aggressive in building the revenue volume in the division. As a result, I am happy to report that we have increased the number of new credit card customers as a result of an aggressive marketing campaign. In addition, we have found some excellent merchandise opportunities. Some of our suppliers have made some of their apparel merchandise avail-able at a deep discount. We have purchased as much of these goods as possible in order to improve profitability. I’m also happy to report that our vendor payment problems have improved. We are nearly caught up on our overdue payables balances.

Comment on the senior vice president’s memo in light of the cash flow information.

Chapter 14 Statement of Cash Flows 693

CP 14-2

CP 14-3

Page 38: Cases & Projects

Analysis of statement of cash flowsDave Chuck is the president and majority shareholder of Xenon Inc., a small retail store chain. Recently, Dave submitted a loan application for Xenon Inc. to Chemistry Bank. It called for a $450,000, 9%, 10-year loan to help finance the construction of a building and the purchase of store equipment, costing a total of $562,500, to enable Xenon Inc. to open a store in Chemistry. Land for this purpose was acquired last year. The bank’s loan officer requested a statement of cash flows in addition to the most recent income statement, balance sheet, and retained earnings statement that Dave had submitted with the loan application.

As a close family friend, Dave asked you to prepare a statement of cash flows. From the records provided, you prepared the following statement:

Xenon Inc. Statement of Cash Flows

For the Year Ended December 31, 2012Cash flows from operating activities:

Net income . . . . . . . . . . . . . . . . . . . . . . . . . . . . . . . . . . . . . . . . . . . . . . . . . . . . $225,000Adjustments to reconcile net income to net cash flow from operating activities:

Depreciation. . . . . . . . . . . . . . . . . . . . . . . . . . . . . . . . . . . . . . . . . . . . . . . . 63,000Gain on sale of investments . . . . . . . . . . . . . . . . . . . . . . . . . . . . . . . . . . (22,500)Changes in current operating assets and liabilities:

Decrease in accounts receivable . . . . . . . . . . . . . . . . . . . . . . . . . . . . 15,750Increase in inventories . . . . . . . . . . . . . . . . . . . . . . . . . . . . . . . . . . . . . (31,500)Increase in accounts payable . . . . . . . . . . . . . . . . . . . . . . . . . . . . . . . 22,500Decrease in accrued expenses payable . . . . . . . . . . . . . . . . . . . . . . (4,500)

Net cash flow from operating activities . . . . . . . . . . . . . . . . . . . . . . . . . . $267,750Cash flows from investing activities:

Cash received from investments sold . . . . . . . . . . . . . . . . . . . . . . . . . . . . $135,000Less cash paid for purchase of store equipment . . . . . . . . . . . . . . . . . . (90,000)Net cash flow provided by investing activities . . . . . . . . . . . . . . . . . . . . 45,000

Cash flows from financing activities:Cash paid for dividends . . . . . . . . . . . . . . . . . . . . . . . . . . . . . . . . . . . . . . . . . $ (94,500)Net cash flow used for financing activities. . . . . . . . . . . . . . . . . . . . . . . . (94,500)

Increase in cash . . . . . . . . . . . . . . . . . . . . . . . . . . . . . . . . . . . . . . . . . . . . . . . . . . . $218,250Cash at the beginning of the year. . . . . . . . . . . . . . . . . . . . . . . . . . . . . . . . . . . 81,000Cash at the end of the year . . . . . . . . . . . . . . . . . . . . . . . . . . . . . . . . . . . . . . . . . $299,250

Schedule of Noncash Financing and Investing Activities:Issued common stock for land $180,000

After reviewing the statement, Dave telephoned you and commented, “Are you sure this statement is right?” Dave then raised the following questions:

1. “How can depreciation be a cash flow?”

2. “Issuing common stock for the land is listed in a separate schedule. This transaction has nothing to do with cash! Shouldn’t this transaction be eliminated from the statement?”

3. “How can the gain on sale of investments be a deduction from net income in deter-mining the cash flow from operating activities?”

4. “Why does the bank need this statement anyway? They can compute the increase in cash from the balance sheets for the last two years.”

After jotting down Dave’s questions, you assured him that this statement was “right.” But to alleviate Dave’s concern, you arranged a meeting for the following day.

a. How would you respond to each of Dave’s questions?

b. Do you think that the statement of cash flows enhances the chances of Xenon Inc. receiving the loan? Discuss.

694 Chapter 14 Statement of Cash Flows

CP 14-4

Page 39: Cases & Projects

Statement of cash flows

Group ProjectThis activity will require two teams to retrieve cash flow statement information from the Internet. One team is to obtain the most recent year’s statement of cash flows for Johnson & Johnson, and the other team the most recent year’s statement of cash flows for AMR Corp.

The statement of cash flows is included as part of the annual report information that is a required disclosure to the Securities and Exchange Commission (SEC). SEC docu-ments can be retrieved using the EdgarScanTM service at http://www.sec.gov/edgar/searchedgar/webusers.htm.

To obtain annual report information, type in a company name in the appropriate space. EdgarScan will list the reports available to you for the company you’ve selected. Select the most recent annual report filing, identified as a 10-K or 10-K405. EdgarScan provides an outline of the report, including the separate financial statements. You can double-click the income statement and balance sheet for the selected company into an ExcelTM spreadsheet for further analysis.

As a group, compare the two statements of cash flows.

a. How are Johnson & Johnson and AMR Corp. similar or different regarding cash flows?

b. Compute and compare the free cash flow for each company, assuming additions to property, plant, and equipment replace current capacity.

F•A•I

Internet Project

Chapter 14 Statement of Cash Flows 695

CP 14-5

Page 40: Cases & Projects

Cases & Projects

Analysis of financing corporate growthAssume that the president of Smokey Mountain Brewery made the following statement in the Annual Report to Shareholders:

“The founding family and majority shareholders of the company do not believe in using debt to finance future growth. The founding family learned from hard experience during Prohibition and the Great Depression that debt can cause loss of flexibility and eventual loss of corporate control. The company will not place itself at such risk. As such, all future growth will be financed either by stock sales to the public or by internally generated resources.”

As a public shareholder of this company, how would you respond to this policy?

Receivables and inventory turnoverLewis Industries, Inc., has completed its fiscal year on December 31, 2012. The auditor, Bill Brewer, has approached the CFO, Rob Beets, regarding the year-end receivables and inventory levels of Lewis Industries. The following conversation takes place:

Bill: We are beginning our audit of Lewis Industries and have prepared ratio analyses to determine if there have been significant changes in operations or financial position. This helps us guide the audit process. This analysis indicates that the inventory turnover has decreased from 5.1 to 2.7, while the accounts receivable turnover has decreased from 11 to 7. I was wondering if you could explain this change in operations.

Rob: There is little need for concern. The inventory represents computers that we were unable to sell during the holiday buying season. We are confident, however, that we will be able to sell these computers as we move into the next fiscal year.

Bill: What gives you this confidence?

Rob: We will increase our advertising and provide some very attractive price concessions to move these machines. We have no choice. Newer technology is already out there, and we have to unload this inventory.

Bill: … and the receivables?

Rob: As you may be aware, the company is under tremendous pressure to expand sales and profits. As a result, we lowered our credit standards to our commercial customers so that we would be able to sell products to a broader customer base. As a result of this policy change, we have been able to expand sales by 35%.

Bill: Your responses have not been reassuring to me.

Rob: I’m a little confused. Assets are good, right? Why don’t you look at our current ratio? It has improved, hasn’t it?I would think that you would view that very favorably.

Why is Bill concerned about the inventory and accounts receivable turnover ratios and Rob’s responses to them? What action may Bill need to take? How would you respond to Rob’s last comment?

Vertical analysisThe condensed income statements through income from operations for Dell Inc. and Apple Inc. are reproduced below for recent fiscal years (numbers in millions of dollars).

Dell Inc. Apple Inc.Sales (net) $61,101 $36,537Cost of sales 50,144 23,397Gross profit $10,957 $13,140Selling, general, and administrative expenses $ 7,104 $ 4,149Research and development 663 1,333Operating expenses $ 7,767 $ 5,482Income from operations $ 3,190 $ 7,658

Prepare comparative common-sized statements, rounding percents to one decimal place. Interpret the analyses.

Chapter 15 Financial Statement Analysis 749

CP 15-1

CP 15-2

CP 15-3

amazzaro
Text Box
Chapter 15
Page 41: Cases & Projects

Profitability and stockholder ratiosHarley-Davidson, lnc., is a leading motorcycle manufacturer in the United States. The com-pany manufactures and sells a number of different types of motorcycles, a complete line of motorcycle parts, and brand-related accessories, clothing, and collectibles. In recent years, Harley-Davidson has attempted to expand its dealer network and product lines internationally.

The following information is available for three recent years (in millions except per-share amounts):

2008 2007 2006Net income (loss) $655 $934 $1,043Preferred dividends $0.00 $0.00 $0.00Interest expense $4.50 $0.00 $0.00Shares outstanding for computing earnings per share 233 238 258Cash dividend per share $1.29 $1.06 $0.81Average total assets $6,743 $5,595 $5,394Average stockholders’ equity $2,246 $2,566 $2,921Average stock price per share $31.29 $58.44 $60.98

1. Calculate the following ratios for each year:

a. Rate earned on total assets

b. Rate earned on stockholders’ equity

c. Earnings per share

d. Dividend yield

e. Price-earnings ratio

2. What is the ratio of average liabilities to average stockholders’ equity for 2008?

3. Explain the direction of the dividend yield and price-earnings ratio in light of Harley-Davidson’s profitability trend.

4. Based on these data, evaluate Harley-Davidson’s strategy to expand to international markets.

Comprehensive profitability and solvency analysisMarriott International, Inc., and Starwood Hotels and Resorts Worldwide, Inc. are two major owners and managers of lodging and resort properties in the United States. Abstracted income state-ment information for the two companies is as follows for a recent year:

Marriott (in millions)

Starwood (in millions)

Operating profit before other expenses and interest $ 815 $ 635Other income (expenses) 57 (95)Interest expense (163) (210)Income before income taxes $ 709 $ 330Income tax expense 350 76Net income $ 359 $ 254

Balance sheet information is as follows:

Marriott (in millions)

Starwood (in millions)

Total liabilities $7,523 $8,082Total stockholders’ equity 1,380 1,621Total liabilities and stockholders’ equity $8,903 $9,703

The average liabilities, stockholders’ equity, and total assets were as follows:

Marriott (in millions)

Starwood (in millions)

Average total liabilities $7,518 $7,814Average total stockholders’ equity 1,405 1,849Average total assets 7,494 9,663

750 Chapter 15 Financial Statement Analysis

CP 15-4

CP 15-5

Page 42: Cases & Projects

1. Determine the following ratios for both companies (round to one decimal place after the whole percent):

a. Rate earned on total assets

b. Rate earned on total stockholders’ equity

c. Number of times interest charges are earned

d. Ratio of liabilities to stockholders’ equity

2. Analyze and compare the two companies, using the information in (1).

Chapter 15 Financial Statement Analysis 751

Page 43: Cases & Projects

Cases & Projects

Ethics and professional conduct in businessJarrett Manufacturing Company allows employees to purchase, at cost, manufacturing materials, such as metal and lumber, for personal use. To purchase materials for personal use, an employee must complete a materials requisition form, which must then be ap-proved by the employee’s immediate supervisor. Felix Thecat, an assistant cost accountant, charges the employee an amount based on Jarrett’s net purchase cost.

Felix Thecat is in the process of replacing a deck on his home and has requisitioned lumber for personal use, which has been approved in accordance with company policy. In computing the cost of the lumber, Felix reviewed all the purchase invoices for the past year. He then used the lowest price to compute the amount due the company for the lumber.

Discuss whether Felix behaved in an ethical manner.

Financial vs. managerial accountingThe following statement was made by the vice president of finance of The Electric Com-pany: “The managers of a company should use the same information as the shareholders of the firm. When managers use the same information in guiding their internal operations as shareholders use in evaluating their investments, the managers will be aligned with the stockholders’ profit objectives.”

Respond to the vice president’s statement.

Managerial accounting in the management processFor each of the following managers, describe how managerial accounting could be used to satisfy strategic or operational objectives:

1. The vice president of the Information Systems Division of a bank.

2. A hospital administrator.

3. The chief executive officer of a food company. The food company is divided into three divisions: Nonalcoholic Beverages, Snack Foods, and Fast Food Restaurants.

4. The manager of the local campus copy shop.

Classifying costsForest & Tree Company provides computer repair services for the community. Jack Dee’s computer was not working, and he called Forest & Tree for a home repair visit. Forest & Tree Company’s technician arrived at 2:00 p.m. to begin work. By 4:00 p.m. the problem was diagnosed as a failed circuit board. Unfortunately, the technician did not have a new circuit board in the truck, since the technician’s previous customer had the same problem, and a board was used on that visit. Replacement boards were available back at Forest & Tree Company’s shop. Therefore, the technician drove back to the shop to retrieve a replacement board. From 4:00 to 5:00 p.m., Forest & Tree Company’s technician drove the round trip to retrieve the replacement board from the shop.

At 5:00 p.m. the technician was back on the job at Jack’s home. The replacement pro-cedure is somewhat complex, since a variety of tests must be performed once the board is installed. The job was completed at 6:00 p.m.

Jack’s repair bill showed the following:

Circuit board $ 85Labor charges 285Total $370

788 Chapter 16 Managerial Accounting Concepts and Principles

CP 16-1

CP 16-2

CP 16-4

CP 16-3

amazzaro
Text Box
Chapter 16
Page 44: Cases & Projects

Jack was surprised at the size of the bill and asked for some greater detail support-ing the calculations. Forest & Tree Company responded with the following explanations:

Cost of materials:Purchase price of circuit board $60Markup on purchase price to cover storage and handling 25Total materials charge $85

The labor charge per hour is detailed as follows:

2:00–3:00 p.m. $ 603:00–4:00 p.m. 504:00–5:00 p.m. 755:00–6:00 p.m. 100Total labor charge $285

Further explanations in the differences in the hourly rates are as follows:

First hour:Base labor rate . . . . . . . . . . . . . . . . . . . . . . . . . . . . . . . . . . . . . . . . . $ 30Fringe benefits . . . . . . . . . . . . . . . . . . . . . . . . . . . . . . . . . . . . . . . . . 12Overhead (other than storage and handling) . . . . . . . . . . . . . 8

Total base labor rate. . . . . . . . . . . . . . . . . . . . . . . . . . . . . . . . . $ 50Additional charge for first hour of any job to cover the cost of vehicle depreciation, fuel, and employee time in transit. A 30-minute transit time is assumed. . . . . . . . . . . . . . 10

$ 60Third hour:

Base labor rate . . . . . . . . . . . . . . . . . . . . . . . . . . . . . . . . . . . . . . . . . $ 50The trip back to the shop includes vehicle depreciation and fuel; therefore, a charge was added to the hourly rate to cover these costs. The round trip took an hour. . . . . . . . . . 25

$ 75Fourth hour:

Base labor rate . . . . . . . . . . . . . . . . . . . . . . . . . . . . . . . . . . . . . . . . . $ 50Overtime premium for time worked in excess of an eight- hour day (starting at 5:00 p.m.) is equal to the base rate. . . . 50

$100

1. If you were in Jack’s position, how would you respond to the bill? Are there parts of the bill that appear incorrect to you? If so, what argument would you employ to convince Forest & Tree Company that the bill is too high?

2. Use the headings below to construct a table. Fill in the table by first listing the costs identified in the activity in the left-hand column. For each cost, place a check mark in the appropriate column identifying the correct cost classification. Assume that each service call is a job.

Cost Direct Materials Direct labor Overhead

using managerial accounting informationThe following situations describe decision scenarios that could use managerial account-ing information:

1. The manager of Barney’s BBQ wishes to determine the price to charge for various lunch plates.

2. By evaluating the cost of leftover materials, the plant manager of a precision tool facility wishes to determine how effectively the plant is being run.

3. The division controller of Quality Plumbing Supplies needs to determine the cost of products left in inventory.

Chapter 16 Managerial Accounting Concepts and Principles 789

CP 16-5

Page 45: Cases & Projects

4. The manager of the Maintenance Department of a large manufacturing company wishes to plan next year’s anticipated expenditures.

For each situation, discuss how managerial accounting information could be used.

Classifying costs

Group ProjectWith a group of students, visit a local copy and graphics shop or a pizza restaurant. As you observe the operation, consider the costs associated with running the business. As a group, identify as many costs as you can and classify them according to the following table headings:

Cost Direct Materials Direct labor Overhead Selling Expenses

790 Chapter 16 Managerial Accounting Concepts and Principles

CP 16-6

Page 46: Cases & Projects

Cases & ProjectsCases & Projects

Managerial analysisThe controller of the plant of Simmons Building Supplies prepared a graph of the unit costs from the job cost reports for Product Alpha. The graph appeared as follows:

Un

it C

ost

$35

$30

$25

$20

$15

$10

$5

$0

Day of WeekDay M T W R F M T W R F M T W R F M T W R F

How would you interpret this information? What further information would you request?

Job order decision making and rate deficienciesJackalope Company makes attachments, such as backhoes and grader and bulldozer blades, for construction equipment. The company uses a job order cost system. Management is concerned about cost performance and evaluates the job cost sheets to learn more about the cost effectiveness of the operations. To facilitate a comparison, the cost sheet for Job 206 (40 backhoe buckets completed in May) was compared with Job 228, which was for 60 backhoe buckets completed in December. The two job cost sheets follow.

Job 206

Item: 40 backhoe buckets

Materials:Direct Materials

Quantity ×Direct Materials

Price = amountSteel (tons) 70 $800 $56,000Steel components (pieces) 420 5 2,100

Total materials $58,100

Direct labor:Direct Labor

hours ×Direct

Labor Rate = amountFoundry 270 $15 $ 4,050Welding 370 18 6,660Shipping 120 12 1,440

Total direct labor 760 $12,150

Direct Total Labor Cost ×

Factory Overhead

Rate = amountFactory overhead

(200% of direct labor dollars) $12,150 × 200% $24,300Total cost $94,550Total units ÷ 40Unit cost (rounded) $ 2,364

Chapter 17 Job Order Costing 829

CP 17-1

CP 17-2CP 17-2

amazzaro
Text Box
Chapter 17
Page 47: Cases & Projects

Job 228

Item: 60 backhoe bucketsDirect Materials

Quantity ×Direct Materials

Price = amountMaterials:

Steel (tons) 150 $780 $117,000Steel components (pieces) 720 5 3,600

Total materials $120,600

Direct Labor hours ×

Direct Labor Rate = amount

Direct labor:Foundry 600 $15 $ 9,000Welding 840 18 15,120Shipping 250 12 3,000

Total direct labor 1,690 $27,120

Direct Total Labor Cost ×

Factory Overhead

Rate = amountFactory overhead

(200% of direct labor dollars) $27,120 × 200% $ 54,420Total cost $201,960Total units ÷ 60Unit cost $ 3,366

Management is concerned with the increase in unit costs over the months from May to December. To understand what has occurred, management interviewed the purchasing manager and quality manager.

Purchasing Manager: Prices have been holding steady for our raw materials during the first half of the year. I found a new supplier for our bulk steel that was willing to offer a better price than we received in the past. I saw these lower steel prices and jumped at them, knowing that a reduction in steel prices would have a very favorable impact on our costs.

Quality Manager: Something happened around mid-year. All of a sudden, we were ex-periencing problems with respect to the quality of our steel. As a result, we’ve been having all sorts of problems on the shop floor in our foundry and welding operation.

1. Analyze the two job cost sheets, and identify why the unit costs have changed for the backhoe buckets. Complete the following schedule to help you in your analysis:

ItemInput Quantity per

Unit—Job 206Input Quantity per

Unit—Job 228SteelFoundry laborWelding labor

2. How would you interpret what has happened in light of your analysis and the interviews?

Factory overhead rateSaltry Inc., a specialized equipment manufacturer, uses a job order costing system. The overhead is allocated to jobs on the basis of direct labor hours. The overhead rate is now $2,000 per direct labor hour. The design engineer thinks that this is illogical. The design engineer has stated the following:

Our accounting system doesn’t make any sense to me. It tells me that every labor hour carries an additional burden of $2,000. This means that direct labor makes up only 5%

830 Chapter 17 Job Order Costing

CP 17-3

Page 48: Cases & Projects

of our total product cost, yet it drives all our costs. In addition, these rates give my design engineers incentives to “design out” direct labor by using machine technology. Yet, over the past years as we have had less and less direct labor, the overhead rate keeps going up and up. I won’t be surprised if next year the rate is $2,500 per direct labor hour. I’m also concerned because small errors in our estimates of the direct labor content can have a large impact on our estimated costs. Just a 30-minute error in our estimate of assembly time is worth $1,000. Small mistakes in our direct labor time estimates really swing our bids around. I think this puts us at a disadvantage when we are going after business.

1. What is the engineer’s concern about the overhead rate going “up and up”?

2. What did the engineer mean about the large overhead rate being a disadvan-tage when placing bids and seeking new business?

3. What do you think is a possible solution?

Recording manufacturing costsSven Olaf just began working as a cost accountant for COLD Industries Inc., which manu-factures gift items. Sven is preparing to record summary journal entries for the month. Sven begins by recording the factory wages as follows:

Wages Expense 50,000Wages Payable 50,000

Then the factory depreciation:

Depreciation Expense—Factory Machinery 15,000Accumulated Depreciation—Factory Machinery 15,000

Sven’s supervisor, Boris Gingrich, walks by and notices the entries. The following conversation takes place:

Boris: That’s a very unusual way to record our factory wages and depreciation for the month.

Sven: What do you mean? This is exactly the way we were taught to record wages and depreciation in school. You know, debit an expense and credit Cash or payables, or in the case of depreciation, credit Accumulated Depreciation.

Boris: Well, it’s not the credits I’m concerned about. It’s the debits—I don’t think you’ve recorded the debits cor-rectly. I wouldn’t mind if you were recording the administrative wages or office equipment depreciation this way, but I’ve got real questions about recording factory wages and factory machinery depreciation this way.

Sven: Now I’m really confused. You mean this is correct for administrative costs, but not for factory costs? Well, what am I supposed to do—and why?

1. Play the role of Boris and answer Sven’s questions.

2. Why would Boris accept the journal entries if they were for administrative costs?

Predetermined overhead ratesAs an assistant cost accountant for Vonn Industries, you have been assigned to review the activity base for the predetermined factory overhead rate. The president, Tony Favre, has expressed concern that the over- or underapplied overhead has fluctuated excessively over the years.

An analysis of the company’s operations and use of the current overhead rate (direct materials usage) has narrowed the possible alternative overhead bases to direct labor cost and machine hours. For the past five years, the following data have been gathered:

2012 2011 2010 2009 2008

Actual overhead $1,062,500 $1,250,000 $1,025,000 $ 975,000 $ 937,500Applied overhead 1,057,500 1,255,000 1,021,000 977,500 938,750(Over-) underapplied overhead $ 5,000 $ (5,000) $ 4,000 $ (2,500) $ (1,250)Direct labor cost $3,560,000 $4,160,000 $3,400,000 $3,240,000 $3,140,000Machine hours 126,250 149,000 122,000 116,000 111,750

Chapter 17 Job Order Costing 831

CP 17-4

CP 17-5

Page 49: Cases & Projects

1. Calculate a predetermined factory overhead rate for each alternative base, assuming that rates would have been determined by relating the total amount of factory overhead for the past five years to the base.

2. For each of the past five years, determine the over- or underapplied overhead, based on the two predetermined overhead rates developed in part (1).

3. Which predetermined overhead rate would you recommend? Discuss the basis for your recommendation.

832 Chapter 17 Job Order Costing

Page 50: Cases & Projects

Cases & Projects

ethics and professional conduct in business

Assume you are the division controller for Auntie M’s Cookie Company. Auntie M has intro-duced a new chocolate chip cookie called Full of Chips, and it is a success. As a result, the product manager responsible for the launch of this new cookie was promoted to division vice president and became your boss. A new product manager, Morgan, has been brought in to replace the promoted manager. Morgan notices that the Full of Chips cookie uses a lot of chips, which increases the cost of the cookie. As a result, Morgan has ordered that the amount of chips used in the cookies be reduced by 10%. The manager believes that a 10% reduction in chips will not adversely affect sales, but will reduce costs, and hence improve margins. The increased margins would help Morgan meet profit targets for the period.

You are looking over some cost of production reports segmented by cookie line. You notice that there is a drop in the materials costs for Full of Chips. On further investiga-tion, you discover why the chip costs have declined (fewer chips). Both you and Morgan report to the division vice president, who was the original product manager for Full of Chips. You are trying to decide what to do, if anything.

Discuss the options you might consider.

Accounting for materials costsIn papermaking operations for companies such as International Paper Company, wet pulp is fed into paper machines, which press and dry pulp into a continuous sheet of paper. The paper is formed at very high speeds (60 mph). Once the paper is formed, the paper is rolled onto a reel at the back end of the paper machine. One of the characteristics of papermaking is the creation of “broke” paper. Broke is paper that fails to satisfy quality standards and is therefore rejected for final shipment to customers. Broke is recycled back to the beginning of the process by combining the recycled paper with virgin (new) pulp material. The combina-tion of virgin pulp and recycled broke is sent to the paper machine for papermaking. Broke is fed into this recycle process continuously from all over the facility.

In this industry, it is typical to charge the papermaking operation with the cost of direct materials, which is a mixture of virgin materials and broke. Broke has a much lower cost than does virgin pulp. Therefore, the more broke in the mixture, the lower the average cost of direct materials to the department. Papermaking managers will frequently comment on the importance of broke for keeping their direct materials costs down.

a. How do you react to this accounting procedure?

b. What “hidden costs” are not considered when accounting for broke as de-scribed above?

Analyzing unit costsContinental Containers Inc. manufactures cans for the canned food industry. The opera-tions manager of a can manufacturing operation wants to conduct a cost study investigating the relationship of tin content in the material (can stock) to the energy cost for enamel-ing the cans. The enameling was necessary to prepare the cans for labeling. A higher percentage of tin content in the can stock increases the cost of material. The operations manager believed that a higher tin content in the can stock would reduce the amount of energy used in enameling. During the analysis period, the amount of tin content in the steel can stock was increased for every month, from April to September. The following operating reports were available from the controller:

Energy

Materials

Total cost

Units produced

Cost per unit

1

2

3

4

5

6

7

April$ 19,500

18,000

$ 37,500

�75,000

$ 0.50

$ 43,200

45,000

$ 88,200

�180,000

$ 0.49

May$ 36,300

42,900

$ 79,200

�165,000

$ 0.48

June$ 21,000

28,350

$ 49,350

�105,000

$ 0.47

July$ 24,300

37,800

$ 62,100

�135,000

$ 0.46

August$ 24,000

43,500

$ 67,500

�150,000

$ 0.45

SeptemberA B C D E F G

880 Chapter 18 Process Cost Systems

CP 18-1

CP 18-2

CP 18-3

amazzaro
Text Box
Chapter 18
Page 51: Cases & Projects

Differences in materials unit costs were entirely related to the amount of tin content.

Interpret this information and report to the operations manager your recommen-dations with respect to tin content.

decision makingDamion Lott, plant manager of Albany Paper Company’s papermaking mill, was looking over the cost of production reports for July and August for the Papermaking Department. The reports revealed the following:

July August

Pulp and chemicals . . . . . . . . . . . . . . . . . . . . . . . . . $295,600 $306,800Conversion cost . . . . . . . . . . . . . . . . . . . . . . . . . . . . 146,000 150,900Total cost . . . . . . . . . . . . . . . . . . . . . . . . . . . . . . . . . . $441,600 $457,700Number of tons . . . . . . . . . . . . . . . . . . . . . . . . . . . . ÷ 1,200 ÷ 1,150Cost per ton . . . . . . . . . . . . . . . . . . . . . . . . . . . . . . . $ 368 $ 398

Damion was concerned about the increased cost per ton from the output of the depart-ment. As a result, he asked the plant controller to perform a study to help explain these results. The controller, Lexi Hammond, began the analysis by performing some interviews of key plant personnel in order to understand what the problem might be. Excerpts from an interview with Chase Wilson, a paper machine operator, follow:

Chase: We have two papermaking machines in the department. I have no data, but I think paper machine No. 1 is applying too much pulp, and thus is wasting both conversion and materials resources. We haven’t had repairs on paper machine No. 1 in a while. Maybe this is the problem.

Lexi: How does too much pulp result in wasted resources?

Chase: Well, you see, if too much pulp is applied, then we will waste pulp material. The customer will not pay for the extra weight. Thus, we just lose that amount of material. Also, when there is too much pulp, the machine must be slowed down in order to complete the drying process. This results in a waste of conversion costs.

Lexi: Do you have any other suspicions?

Chase: Well, as you know, we have two products—green paper and yellow paper. They are identical except for the color. The color is added to the papermaking process in the paper machine. I think that during August these two color papers have been behaving very differently. I don’t have any data, but it just seems as though the amount of waste associated with the green paper has increased.

Lexi: Why is this?

Chase:Iunderstandthattherehasbeenachangeinspecificationsforthegreenpaper,startingnearthebeginningof August. This change could be causing the machines to run poorly when making green paper. If this is the case, the cost per ton would increase for green paper.

Lexi also asked for a database printout providing greater detail on August’s operating results.

September 9 Requested by: Lexi Hammond

Papermaking Department—August detail

1

2

3

4

5

6

7

8

1

2

3

4

5

6

7

8

9

10

11

12

13

Paper Production

Run Number Machine

Material Costs

Conversion Costs Tons Color

1

1

1

1

2

2

2

2

Total

Green

Yellow

Green

Yellow

Green

Yellow

Green

Yellow

41,250

41,700

44,600

36,100

38,300

35,650

35,600

33,600

306,800

19,100

21,200

22,500

18,100

18,900

15,700

18,400

17,000

150,900

160

140

150

120

160

150

130

140

1,150

A B C D E F

Assuming that you’re Lexi Hammond, write a memo to Damion Lott with a recommenda-tion to management. You should analyze the August data to determine whether the paper machine or the paper color explains the increase in the unit cost from July. Include any supporting schedules that are appropriate.

Chapter 18 Process Cost Systems 881

CP 18-4

Page 52: Cases & Projects

Process costing companies

group ProjectThe following categories represent typical process manufacturing industries:

Beverages MetalsChemicals Petroleum refiningFood PharmaceuticalsForest and paper products Soap and cosmetics

In groups of two or three, for each category identify one company (following your in-structor’s specific instructions) and determine the following:

1. Typical products manufactured by the selected company, including brand names.

2. Typical raw materials used by the selected company.

3. Types of processes used by the selected company.

Use annual reports, the Internet, or library resources in doing this activity.

Internet Project

882 Chapter 18 Process Cost Systems

CP 18-5

Page 53: Cases & Projects

Cases & Projects

Ethics and professional conduct in business

Carrie Skilling is a financial consultant to Ron En Trust Inc., a real estate syndicate. Ron En Trust Inc. finances and develops commercial real estate (office buildings). The completed projects are then sold as limited partnership interests to individual investors. The syndicate makes a profit on the sale of these partnership interests. Carrie provides financial information for the offering prospectus, which is a document that provides the financial and legal details of the limited partnership offerings. In one of the projects, the bank has financed the construction of a commercial office building at a rate of 10% for the first four years, after which time the rate jumps to 15% for the remaining 20 years of the mortgage. The interest costs are one of the major ongoing costs of a real estate project. Carrie has reported prominently in the pro-spectus that the break-even occupancy for the first four years is 65%. This is the amount of office space that must be leased to cover the interest and general upkeep costs over the first four years. The 65% break-even is very low and thus communicates a low risk to potential investors. Carrie uses the 65% break-even rate as a major marketing tool in selling the limited partnership interests. Buried in the fine print of the prospectus is additional information that would allow an astute investor to determine that the break-even occupancy will jump to 95% after the fourth year because of the contracted increase in the mortgage interest rate. Carrie believes prospective investors are adequately informed as to the risk of the investment.

Comment on the ethical considerations of this situation.

Break-even sales, contribution margin“For a student, a grade of 65 percent is nothing to write home about. But for the airline . . . [industry], filling 65 percent of the seats . . . is the difference between profit and loss.

The [economy] might be just strong enough to sustain all the carriers on a cash basis, but not strong enough to bring any significant profitability to the industry. . . . For the air-lines . . ., the emphasis will be on trying to consolidate routes and raise ticket prices. . . .”

The airline industry is notorious for boom and bust cycles. Why is airline profit-ability very sensitive to these cycles? Do you think that during a down cycle the strategy to consolidate routes and raise ticket prices is reasonable? What would make this strategy succeed or fail? Why?

Source: Edwin McDowell, “Empty Seats, Empty Beds, Empty Pockets,” The New York Times, January 6, 1992, p. C3.

Break-even analysisTimbuk 3 Inc. has finished a new video game, Snowboard Challenge. Management is now considering its marketing strategies. The following information is available:

Anticipated sales price per unit . . . . . . . . . . . . . . . . . . . . . . $50Variable cost per unit* . . . . . . . . . . . . . . . . . . . . . . . . . . . . . . . $10Anticipated volume . . . . . . . . . . . . . . . . . . . . . . . . . . . . . . . . . 500,000 unitsProduction costs . . . . . . . . . . . . . . . . . . . . . . . . . . . . . . . . . . . . $14,000,000Anticipated advertising . . . . . . . . . . . . . . . . . . . . . . . . . . . . . . $6,000,000

*The cost of the video game, packaging, and copying costs.

Two managers, Michele Woodard and Mark Hobson, had the following discussion of ways to increase the profitability of this new offering:

Michele: I think we need to think of some way to increase our profitability. Do you have any ideas?

Mark: Well, I think the best strategy would be to become aggressive on price.

Michele: How aggressive?

Mark: If we drop the price to $35 per unit and maintain our advertising budget at $6,000,000, I think we will gener-ate sales of 1,000,000 units.

926 Chapter 19 Cost Behavior and Cost-Volume-Profit Analysis

CP 19-1

CP 19-2

CP 19-3

amazzaro
Text Box
Chapter 19
Page 54: Cases & Projects

Michele: I think that’s the wrong way to go. You’re giving too much up on price. Instead, I think we need to follow an aggressive advertising strategy.

Mark: How aggressive?

Michele: If we increase our advertising to a total of $8,000,000, we should be able to increase sales volume to 750,000 units without any change in price.

Mark: I don’t think that’s reasonable. We’ll never cover the increased advertising costs.

Which strategy is best: Do nothing? Follow the advice of Mark Hobson? Or fol-low Michele Woodard’s strategy?

Variable costs and activity bases in decision makingThe owner of Varsity Printing, a printing company, is planning direct labor needs for the upcoming year. The owner has provided you with the following information for next year’s plans:

One Color Two Color Three Color Four Color Total

Number of banners 106 137 308 349 900

Each color on the banner must be printed one at a time. Thus, for example, a four-color banner will need to be run through the printing operation four separate times. The total production volume last year was 400 banners, as shown below.

One Color Two Color Three Color Total

Number of banners 90 120 190 400

As you can see, the four-color banner is a new product offering for the upcoming year. The owner believes that the expected 500-unit increase in volume from last year means that direct labor expenses should increase by 125% (500/400). What do you think?

Variable costs and activity bases in decision makingSales volume has been dropping at Textron Company. During this time, however, the Shipping Department manager has been under severe financial constraints. The manager knows that most of the Shipping Department’s effort is related to pulling inventory from the warehouse for each order and performing the paperwork. The paperwork involves preparing shipping documents for each order. Thus, the pulling and paperwork effort associated with each sales order is essentially the same, regardless of the size of the or-der. The Shipping Department manager has discussed the financial situation with senior management. Senior management has responded by pointing out that sales volume has been dropping, so that the amount of work in the Shipping Department should be drop-ping. Thus, senior management told the Shipping Department manager that costs should be decreasing in the department.

The Shipping Department manager prepared the following information:

Month

Sales Volume

Number of Customer Orders

Sales Volume per Order

January $279,000 900 310February 280,800 936 300March 270,750 950 285April 253,500 975 260May 275,000 1,100 250June 249,600 1,040 240July 245,180 1,066 230August 253,125 1,125 225

Given this information, how would you respond to senior management?

Chapter 19 Cost Behavior and Cost-Volume-Profit Analysis 927

CP 19-4

CP 19-5

Page 55: Cases & Projects

Break-even analysis

Group ProjectBreak-even analysis is one of the most fundamental tools for managing any kind of business unit. Consider the management of your school. In a group, brainstorm some applications of break-even analysis at your school. Identify three areas where break-even analysis might be used. For each area, identify the revenues, variable costs, and fixed costs that would be used in the calculation.

928 Chapter 19 Cost Behavior and Cost-Volume-Profit Analysis

CP 19-6

Page 56: Cases & Projects

Data for each style should be reported through contribution margin. The fixed costs should be deducted from the total contribution margin as reported in the “Total” column. For purposes of this problem, the additional expenditure of $113,400 for the assistant brand manager’s salary can be added to the fixed operating expenses.

4. By how much would total annual income increase above its present level if Proposal 3 is accepted? Explain.

Contribution margin analysisLawrence Company manufactures only one product. For the year ended December 31, 2012, the contribution margin decreased by $87,500 from the planned level of $375,000. The president of Lawrence Company has expressed some concern about this decrease and has requested a follow-up report.

The following data have been gathered from the accounting records for the year ended December 31, 2012:

Actual Planned

Difference— Increase or (Decrease)

Sales . . . . . . . . . . . . . . . . . . . . . . . . . . . . . . . . . . . . . . . . . . . . . . . . . . . . . $ 1,581,250 $1,437,500 $143,750Less: Variable cost of goods sold . . . . . . . . . . . . . . . . . . . . . . . . . . . . . . $ 718,750 $ 687,500 $ 31,250 Variable selling and administrative expenses . . . . . . . . . . . . . 575,000 375,000 200,000 Total . . . . . . . . . . . . . . . . . . . . . . . . . . . . . . . . . . . . . . . . . . . . . . . . . $1,293,750 $1,062,500 $231,250Contribution margin . . . . . . . . . . . . . . . . . . . . . . . . . . . . . . . . . . . . . . $ 287,500 $ 375,000 $ (87,500)

Number of units sold . . . . . . . . . . . . . . . . . . . . . . . . . . . . . . . . . . . . . . 28,750 25,000Per unit: Sales price . . . . . . . . . . . . . . . . . . . . . . . . . . . . . . . . . . . . . . . . . . . . . . $55.00 $57.50 Variable cost of goods sold . . . . . . . . . . . . . . . . . . . . . . . . . . . . . . 25.00 27.50 Variable selling and administrative expenses . . . . . . . . . . . . 20.00 15.00

Instructions1. Prepare a contribution margin analysis report for the year ended December 31, 2012.

2. At a meeting of the board of directors on January 30, 2013, the president, after reviewing the contribution margin analysis report, made the following comment:

“It looks as if the price decrease of $2.50 had the effect of increasing sales. However, we lost control over the variable cost of goods sold and variable selling and administrative expenses. Let’s look into these expenses and get them under control! Also, let’s consider decreasing the sales price to $50 to increase sales further.”Do you agree with the president’s comment? Explain.

Cases & Projects

Ethics and professional conduct in businessThe Northeast Division of Confield Inc. uses absorption costing for profit reporting. The general manager of the Northeast Division is concerned about meeting the income objectives of the division. At the beginning of the reporting period, the division had an adequate supply of inventory. The general manager has decided to increase production of goods in the plant in order to allocate fixed manufacturing cost over a greater number of units. Unfortunately, the increased production cannot be sold and will increase the inventory. However, the impact on earnings will be positive because the lower cost per unit will be matched against sales. The general manager has come to Palko Martin, the controller, to determine exactly how much additional production is required in order to increase net income enough to meet the division’s profit objectives. Martin analyzes the data and determines that the inventory will need to be increased by 30% in order to ab-sorb enough fixed costs and meet the income objective. Martin reports this information to the division manager.

Discuss whether Martin is acting in an ethical manner.

ObJ. 5

1. Sales quantity factor, $215,625

Chapter 20 Variable Costing for Management Analysis 973 Chapter 20 Variable Costing for Management Analysis 973

PR 20-6B

CP 20-1

amazzaro
Text Box
Chapter 20
Page 57: Cases & Projects

Inventories under absorption costingMcCray, Inc. manufactures control panels for the electronics industry and has just com-pleted its first year of operations. The following discussion took place between the con-troller, Charles Roehm, and the company president, Michelle Iacovou:

Michelle: I’ve been looking over our first year’s performance by quarters. Our earnings have been increasing each quarter, even though our sales have been flat and our prices and costs have not changed. Why is this?

Charles: Our actual sales have stayed even throughout the year, but we’ve been increasing the utilization of our fac-tory every quarter. By keeping our factory utilization high, we will keep our costs down by allocating the fixed plant costs over a greater number of units. Naturally, this causes our cost per unit to be lower than it would be otherwise.

Michelle: Yes, but what good is this if we have been unable to sell everything that we make? Our inventory is also increasing.

Charles: This is true. However, our unit costs are lower because of the additional production. When these lower costs are matched against sales, it has a positive impact on our earnings.

Michelle: Are you saying that we are able to create additional earnings merely by building inventory? Can this be true?

Charles: Well, I’ve never thought about it quite that way . . . but I guess so.

Michelle: And another thing. What will happen if we begin to reduce our production in order to liquidate the inven-tory? Don’t tell me our earnings will go down even though our production effort drops!

Charles: Well . . .

Michelle: There must be a better way. I’d like our quarterly income statements to reflect what’s really going on. I don’t want our income reports to reward building inventory and penalize reducing inventory.

Charles: I’m not sure what I can do—we have to follow generally accepted accounting principles.

1. Why does reporting income under generally accepted accounting principles “reward” building inventory and “penalize” reducing inventory?

2. What advice would you give to Charles in responding to Michelle’s concern about the present method of profit reporting?

Segmented contribution margin analysisHolly Inc. manufactures and sells devices used in cardiovascular surgery. The company has two salespersons, Bruce and Lewis.

A contribution margin by salesperson report was prepared as follows:

Holly Inc. Contribution Margin by Salesperson

Bruce Lewis

Sales . . . . . . . . . . . . . . . . . . . . . . . . . . . . . . . . . . . . . . . . . . . . . . . . . . . . . . . . . . . . . . . . . . . . . $250,000 $300,000Variable cost of goods sold . . . . . . . . . . . . . . . . . . . . . . . . . . . . . . . . . . . . . . . . . . . . . . . . $ 87,500 $165,000Manufacturing margin . . . . . . . . . . . . . . . . . . . . . . . . . . . . . . . . . . . . . . . . . . . . . . . . . . . . $162,500 $135,000Variable promotion expenses . . . . . . . . . . . . . . . . . . . . . . . . . . . . . . . . . . . . . . . . . . . . . . $ 62,500 $ 15,000Variable sales commission expenses . . . . . . . . . . . . . . . . . . . . . . . . . . . . . . . . . . . . . . . 37,500 45,000

$100,000 $ 60,000Contribution margin . . . . . . . . . . . . . . . . . . . . . . . . . . . . . . . . . . . . . . . . . . . . . . . . . . . . . . $ 62,500 $ 75,000Manufacturing margin as a percent of sales (manufacturing margin ratio) . . . . . . . . . . . . . . . . . . . . . . . . . . . . . . . . . . . . . . . . . . . . 65% 45%Contribution margin ratio . . . . . . . . . . . . . . . . . . . . . . . . . . . . . . . . . . . . . . . . . . . . . . . . . 25% 25%

Interpret the report, and provide recommendations to the two salespersons for improving profitability.

Margin analysisWesterman Equipment Inc. manufactures and sells kitchen cooking products throughout the state. The company employs four salespersons. The following contribution margin by salesperson analysis was prepared:

974 Chapter 20 Variable Costing for Management Analysis974 Chapter 20 Variable Costing for Management Analysis

CP 20-2

CP 20-3

CP 20-4

Page 58: Cases & Projects

Westerman Equipment Inc. Contribution Margin Analysis by Salesperson

Beatty Canace Herbst Saladin

Sales . . . . . . . . . . . . . . . . . . . . . . . . . . . . . . . . . . $150,000 $170,000 $160,000 $ 120,000Variable cost of goods sold . . . . . . . . . . . . . 48,000 81,600 76,800 57,600Manufacturing margin . . . . . . . . . . . . . . . . . $102,000 $ 88,400 $ 83,200 $ 62,400Variable selling expenses Commissions . . . . . . . . . . . . . . . . . . . . . . . . $ 7,500 $ 8,500 $ 8,000 $ 6,000 Promotion expenses . . . . . . . . . . . . . . . . . 46,500 45,900 43,200 32,400 Total variable selling expenses . . . . . $ 54,000 $ 54,400 $ 51,200 $ 38,400Contribution margin . . . . . . . . . . . . . . . . . . . $ 48,000 $ 34,000 $ 32,000 $ 24,000

1. Calculate the manufacturing margin as a percent of sales and the contribution margin ratio for each salesperson.

2. Explain the results of the analysis.

Contribution margin analysisKing Industrial Supply Company sells artistic supplies to retailers in three different states—Arizona, California, and Nevada. The following profit analysis by state was prepared by the company:

Arizona California Nevada

Revenue $900,000 $787,500 $945,000Cost of goods sold 450,000 427,500 450,000Gross profit $450,000 $360,000 $495,000Selling expenses 292,500 270,000 337,500Income from operations $157,500 $ 90,000 $157,500

The following fixed costs have also been provided:

Arizona California Nevada

Fixed manufacturing costs $90,000 $180,000 $101,250Fixed selling expenses 67,500 108,000 90,900

In addition, assume that inventories have been negligible. Management believes it could increase state sales by 20%, without increasing any of

the fixed costs, by spending an additional $33,750 per state on advertising.

1. Prepare a contribution margin by state report for King Industrial Supply Company.

2. Determine how much state operating profit will be generated for an additional $33,750 per state on advertising.

3. Which state will provide the greatest profit return for a $33,750 increase in advertis-ing? Why?

Absorption costing

Group ProjectPatel Company is a family-owned business in which you own 20% of the common stock and your brothers and sisters own the remaining shares. The employment contract of Patel’s new president, Ram Moss, stipulates a base salary of $128,000 per year plus 10% of income from operations in excess of $340,000. Moss uses the absorption costing method of reporting income from operations, which has averaged approximately $480,000 for the past several years.

Sales for 2012, Moss’s first year as president of Patel Company, are estimated at 40,000 units at a selling price of $96 per unit. To maximize the use of Patel’s productive capacity, Moss has decided to manufacture 50,000 units, rather than the 40,000 units of estimated sales. The beginning inventory at January 1, 2012, is insignificant in amount, and the manufacturing costs and selling and administrative expenses for the production of 40,000 and 50,000 units are as follows:

Chapter 20 Variable Costing for Management Analysis 975 Chapter 20 Variable Costing for Management Analysis 975

CP 20-5

CP 20-6

Page 59: Cases & Projects

40,000 Units to Be Manufactured

Number of Units

Unit Cost Total Cost

Manufacturing costs: Variable . . . . . . . . . . . . . . . . . . . . . . . . . . . . . . . . . . . . . . . . . . . . 40,000 $45 $1,800,000 Fixed . . . . . . . . . . . . . . . . . . . . . . . . . . . . . . . . . . . . . . . . . . . . . . . 40,000 10 400,000 Total . . . . . . . . . . . . . . . . . . . . . . . . . . . . . . . . . . . . . . . . . . . . . $55 $2,200,000Selling and administrative expenses: Variable . . . . . . . . . . . . . . . . . . . . . . . . . . . . . . . . . . . . . . . . . . . . $ 960,000 Fixed . . . . . . . . . . . . . . . . . . . . . . . . . . . . . . . . . . . . . . . . . . . . . . . 320,000 Total . . . . . . . . . . . . . . . . . . . . . . . . . . . . . . . . . . . . . . . . . . . . . $1,280,000

50,000 Units to Be Manufactured

Number of Units

Unit Cost Total Cost

Manufacturing costs: Variable . . . . . . . . . . . . . . . . . . . . . . . . . . . . . . . . . . . . . 50,000 $45 $2,250,000 Fixed . . . . . . . . . . . . . . . . . . . . . . . . . . . . . . . . . . . . . . . . 50,000 8 400,000 Total . . . . . . . . . . . . . . . . . . . . . . . . . . . . . . . . . . . . . . $53 $2,650,000Selling and administrative expenses: Variable . . . . . . . . . . . . . . . . . . . . . . . . . . . . . . . . . . . . . $ 960,000 Fixed . . . . . . . . . . . . . . . . . . . . . . . . . . . . . . . . . . . . . . . . 320,000 Total . . . . . . . . . . . . . . . . . . . . . . . . . . . . . . . . . . . . . . $1,280,000

1. In one group, prepare an absorption costing income statement for the year ending December 31, 2012, based on sales of 40,000 units and the manufacture of 40,000 units. In the other group, conduct the same analysis, assuming production of 50,000 units.

2. Explain the difference in the income from operations reported in (1).

3. Compute Moss’s total salary for the year 2012, based on sales of 40,000 units and the manufacture of 40,000 units (Group 1) and 50,000 units (Group 2). Compare your answers.

4. In addition to maximizing the use of Patel Company’s productive capacity, why might Moss wish to manufacture 50,000 units rather than 40,000 units?

5. Can you suggest an alternative way in which Moss’s salary could be deter-mined, using a base salary of $128,000 and 10% of income from operations in excess of $340,000, so that the salary could not be increased by simply manufacturing more units?

40,000 Units to Be Manufactured

Number of Units

Unit Cost Total Cost

Manufacturing costs: Variable . . . . . . . . . . . . . . . . . . . . . . . . . . . . . . . . . . . . . . . . . . . . 40,000 $45 $1,800,000 Fixed . . . . . . . . . . . . . . . . . . . . . . . . . . . . . . . . . . . . . . . . . . . . . . . 40,000 10 400,000 Total . . . . . . . . . . . . . . . . . . . . . . . . . . . . . . . . . . . . . . . . . . . . . $55 $2,200,000Selling and administrative expenses: Variable . . . . . . . . . . . . . . . . . . . . . . . . . . . . . . . . . . . . . . . . . . . . $ 960,000 Fixed . . . . . . . . . . . . . . . . . . . . . . . . . . . . . . . . . . . . . . . . . . . . . . . 320,000 Total . . . . . . . . . . . . . . . . . . . . . . . . . . . . . . . . . . . . . . . . . . . . . $1,280,000

50,000 Units to Be Manufactured

Number of Units

Unit Cost Total Cost

Manufacturing costs: Variable . . . . . . . . . . . . . . . . . . . . . . . . . . . . . . . . . . . . . 50,000 $45 $2,250,000 Fixed . . . . . . . . . . . . . . . . . . . . . . . . . . . . . . . . . . . . . . . . 50,000 8 400,000 Total . . . . . . . . . . . . . . . . . . . . . . . . . . . . . . . . . . . . . . $53 $2,650,000Selling and administrative expenses: Variable . . . . . . . . . . . . . . . . . . . . . . . . . . . . . . . . . . . . . $ 960,000 Fixed . . . . . . . . . . . . . . . . . . . . . . . . . . . . . . . . . . . . . . . . 320,000 Total . . . . . . . . . . . . . . . . . . . . . . . . . . . . . . . . . . . . . . $1,280,000

1. In one group, prepare an absorption costing income statement for the year ending December 31, 2012, based on sales of 40,000 units and the manufacture of 40,000 units. In the other group, conduct the same analysis, assuming production of 50,000 units.

2. Explain the difference in the income from operations reported in (1).

3. Compute Moss’s total salary for the year 2012, based on sales of 40,000 units and the manufacture of 40,000 units (Group 1) and 50,000 units (Group 2). Compare your answers.

4. In addition to maximizing the use of Patel Company’s productive capacity, why might Moss wish to manufacture 50,000 units rather than 40,000 units?

5. Can you suggest an alternative way in which Moss’s salary could be deter-mined, using a base salary of $128,000 and 10% of income from operations in excess of $340,000, so that the salary could not be increased by simply manufacturing more units?

976 Chapter 20 Variable Costing for Management Analysis976 Chapter 20 Variable Costing for Management Analysis

Page 60: Cases & Projects

Cases & Projects

Budget estimates of manufacturing costs and operating expenses for the year are summarized as follows:

estimated Costs and expenses

Fixed (Total for Year)

Variable (Per unit Sold)

Cost of goods manufactured and sold:Direct materials . . . . . . . . . . . . . . . . . . . . . . . . . . . . . . . . . . . . . . . . . — $25.00Direct labor . . . . . . . . . . . . . . . . . . . . . . . . . . . . . . . . . . . . . . . . . . . . . — 7.80Factory overhead:

Depreciation of plant and equipment. . . . . . . . . . . . . . . . . . . $ 3,200 —Other factory overhead . . . . . . . . . . . . . . . . . . . . . . . . . . . . . . . . 1,100 4.50

Selling expenses:Sales salaries and commissions . . . . . . . . . . . . . . . . . . . . . . . . . . . 10,500 12.80Advertising . . . . . . . . . . . . . . . . . . . . . . . . . . . . . . . . . . . . . . . . . . . . . 11,800 —Miscellaneous selling expense . . . . . . . . . . . . . . . . . . . . . . . . . . . 900 2.00

Administrative expenses:Office and officers salaries . . . . . . . . . . . . . . . . . . . . . . . . . . . . . . . 7,400 6.25Supplies . . . . . . . . . . . . . . . . . . . . . . . . . . . . . . . . . . . . . . . . . . . . . . . . 400 1.00Miscellaneous administrative expense . . . . . . . . . . . . . . . . . . . . 200 1.50

Balances of accounts receivable, prepaid expenses, and accounts payable at the end of the year are not expected to differ significantly from the beginning balances. Federal income tax of $26,000 on 2013 taxable income will be paid during 2013. Regular quarterly cash dividends of $0.15 per share are expected to be declared and paid in March, June, September, and December on 30,000 shares of common stock outstanding. It is anticipated that fixed assets will be purchased for $17,500 cash in May.

instructions1. Prepare a budgeted income statement for 2013.

2. Prepare a budgeted balance sheet as of December 31, 2013, with supporting calculations.

ethics and professional conduct in businessThe director of marketing for Starr Computer Co., Sandi Keller, had the following discus-sion with the company controller, Tyler Johnson, on July 26 of the current year:

Sandi: Tyler, it looks like I’m going to spend much less than indicated on my July budget.

Tyler: I’m glad to hear it.

Sandi: Well, I’m not so sure it’s good news. I’m concerned that the president will see that I’m under budget and reduce my budget in the future. The only reason that I look good is that we’ve delayed an advertising campaign. Once the campaign hits in September, I’m sure my actual expenditures will go up. You see, we are also having our sales convention in September. Having the advertising campaign and the convention at the same time is going to kill my September numbers.

Tyler: I don’t think that’s anything to worry about. We all expect some variation in actual spending month to month. What’s really important is staying within the budgeted targets for the year. Does that look as if it’s going to be a problem?

Sandi: I don’t think so, but just the same, I’d like to be on the safe side.

Tyler: What do you mean?

Sandi: Well, this is what I’d like to do. I want to pay the convention-related costs in advance this month. I’ll pay the hotel for room and convention space and purchase the airline tickets in advance. In this way, I can charge all these expenditures to July’s budget. This would cause my actual expenses to come close to budget for July. Moreover, when the big advertising campaign hits in September, I won’t have to worry about expenditures for the convention on my September budget as well. The convention costs will already be paid. Thus, my September expenses should be pretty close to budget.

Tyler: I can’t tell you when to make your convention purchases, but I’m not too sure that it should be expensed on July’s budget.

Chapter 21 Budgeting 1021

CP 21-1

amazzaro
Text Box
Chapter 21
Page 61: Cases & Projects

Sandi: What’s the problem? It looks like “no harm, no foul” to me. I can’t see that there’s anything wrong with this—it’s just smart management.

How should Tyler Johnson respond to Sandi Keller’s request to expense the advanced payments for convention-related costs against July’s budget?

evaluating budgeting systemsChildren’s Hospital of the King’s Daughters Health System in Norfolk, Virginia, introduced a new budgeting method that allowed the hospital’s annual plan to be updated for changes in operating plans. For example, if the budget was based on 400 patient-days (number of patients × number of days in the hospital) and the actual count rose to 450 patient-days, the variable costs of staffing, lab work, and medication costs could be adjusted to reflect this change. The budget manager stated, “I work with hospital directors to turn data into meaningful information and effect change before the month ends.”

a. What budgeting methods are being used under the new approach?

b. Why are these methods superior to the former approaches?

Service company static decision makingA bank manager of City Savings Bank Inc. uses the managerial accounting system to track the costs of operating the various departments within the bank. The departments include Cash Management, Trust, Commercial Loans, Mortgage Loans, Operations, Credit Card, and Branch Services. The budget and actual results for the Operations Department are as follows:

resources Budget actualSalaries $200,000 $200,000Benefits 30,000 30,000Supplies 45,000 42,000Travel 20,000 30,000Training 25,000 35,000Overtime 25,000 20,000Total $345,000 $357,000Excess of actual over budget $ 12,000

a. What information is provided by the budget? Specifically, what questions can the bank manager ask of the Operations Department manager?

b. What information does the budget fail to provide? Specifically, could the bud-get information be presented differently to provide even more insight for the bank manager?

Objectives of the master budgetDomino’s Pizza L.L.C. operates pizza delivery and carry-out restaurants. The annual report describes its business as follows:

We offer a focused menu of high-quality, value-priced pizza with three types of crust (Hand-Tossed, Thin Crust, and Deep Dish), along with buffalo wings, bread sticks, cheesy bread, CinnaStix®, and Coca-Cola® products. Our hand-tossed pizza is made from fresh dough produced in our regional distribution centers. We prepare every pizza using real cheese, pizza sauce made from fresh tomatoes, and a choice of high-quality meat and vegetable toppings in generous portions. Our focused menu and use of premium ingredients enable us to consistently and efficiently produce the highest-quality pizza.

Over the 41 years since our founding, we have developed a simple, cost-efficient model. We offer a limited menu, our stores are designed for delivery and carry-out, and we do not generally offer dine-in service. As a result, our stores require relatively small, lower-rent locations and limited capital expenditures.

How would a master budget support planning, directing, and control for Domino’s?

1022 Chapter 21 Budgeting

CP 21-2

CP 21-3

CP 21-4

Page 62: Cases & Projects

integrity and evaluating budgeting systemsThe city of West Plaines has an annual budget cycle that begins on July 1 and ends on June 30. At the beginning of each budget year, an annual budget is established for each department. The annual budget is divided by 12 months to provide a constant monthly static budget. On June 30, all unspent budgeted monies for the budget year from the various city departments must be “returned” to the General Fund. Thus, if department heads fail to use their budget by year-end, they will lose it. A budget analyst prepared a chart of the difference between the monthly actual and budgeted amounts for the recent fiscal year. The chart was as follows:

Do

llars

Months

Actual Less Budget

(15,000)

(10,000)

(5,000)

0

5,000

10,000

15,000

20,000

25,000

30,000

$35,000

Janu

ary

Febr

uary

Mar

ch

Apr

il

May

June

Dec

embe

r

Nov

embe

r

Oct

ober

Sept

embe

r

Aug

ust

July

a. Interpret the chart.

b. Suggest an improvement in the budget system.

Budget for a state government

Group ProjectIn a group, find the home page of the state in which you presently live. The home page will be of the form www.statename.gov. For example, the state of Tennessee would be found at www.tennessee.gov. At the home page site, search for annual budget information.

1. What are the budgeted sources of revenue and their percentage breakdown?

2. What are the major categories of budgeted expenditures (or appropriations) and their percentage breakdown?

3. Is the projected budget in balance?

Internet Project

Chapter 21 Budgeting 1023

CP 21-5

CP 21-6

Page 63: Cases & Projects

Cases & projects

ethics and professional conduct in business using nonmanufacturing standardsMackenzie Adamson is a cost analyst with Madison Insurance Company. Madison is apply-ing standards to its claims payment operation. Claims payment is a repetitive operation that could be evaluated with standards. Mackenzie used time and motion studies to identify an ideal standard of 36 claims processed per hour. The Claims Processing Department manager, Jonathan Hall, has rejected this standard and has argued that the standard should be 30 claims processed per hour. Jonathan and Mackenzie were unable to agree, so they decided to discuss this matter openly at a joint meeting with the vice president of operations, who would arbitrate a final decision. Prior to the meeting, Mackenzie wrote the following memo to the VP.

To: Linda Clark, Vice President of OperationsFrom: Mackenzie Adamson Re: Standards in the Claims Processing Department

As you know, Jonathan and I are scheduled to meet with you to discuss our disagree-ment with respect to the appropriate standards for the Claims Processing Department. I have conducted time and motion studies and have determined that the ideal standard is 36 claims processed per hour. Jonathan argues that 30 claims processed per hour would be more appropriate. I believe he is trying to “pad” the budget with some slack. I’m not sure what he is trying to get away with, but I believe a tight standard will drive efficiency up in his area. I hope you will agree when we meet with you next week.

Discuss the ethical and professional issues in this situation.

nonfinancial performance measuresThe senior management of Caldwell Company has proposed the following three perfor-mance measures for the company:

1. Net income as a percent of stockholders’ equity

2. Revenue growth

3. Employee satisfaction

Management believes these three measures combine both financial and nonfinancial measures and are thus superior to using just financial measures.

What advice would you give Caldwell Company for improving its performance measurement system?

variance interpretationYou have been asked to investigate some cost problems in the Assembly Department of Elgin Electronics Co., a consumer electronics company. To begin your investigation, you have obtained the following budget performance report for the department for the last quarter:

elgin electronics Co.—assembly department Quarterly Budget performance Report

Standard Quantity at

Standard Rates

actual Quantity at Standard

RatesQuantity variances

Direct labor $ 78,750 $113,750 $35,000 UDirect materials 148,750 192,500 43,750 U

Total $227,500 $306,250 $78,750 U

The following reports were also obtained:

1068 Chapter 22 Performance Evaluation Using Variances from Standard Costs

Cp 22-1

Cp 22-2

Cp 22-3

amazzaro
Text Box
Chapter 22
Page 64: Cases & Projects

elgin electronics Co.—purchasing department Quarterly Budget performance Report

actualQuantity at

Standard Rates

actualQuantity at

actual Ratesprice

varianceDirect materials $218,750 $192,500 –$26,250 F

elgin electronics Co.—fabrication departmentQuarterly Budget performance Report

StandardQuantity at

Standard Rates

actualQuantity at

Standard RatesQuantityvariances

Direct labor $122,500 $101,500 –$21,000 FDirect materials 70,000 70,000 0

Total $192,500 $171,500 –$21,000 F

You also interviewed the Assembly Department supervisor. Excerpts from the interview follow.

Q: What explains the poor performance in your department?A: Listen, you’ve got to understand what it’s been like in this department recently. Lately,

it seems no matter how hard we try, we can’t seem to make the standards. I’m not sure what is going on, but we’ve been having a lot of problems lately.

Q: What kind of problems?A: Well, for instance, all this quarter we’ve been requisitioning purchased parts from

the material storeroom, and the parts just didn’t fit together very well. I’m not sure what is going on, but during most of this quarter we’ve had to scrap and sort pur-chased parts—just to get our assemblies put together. Naturally, all this takes time and material. And that’s not all.

Q: Go on.A: All this quarter, the work that we’ve been receiving from the Fabrication Depart-

ment has been shoddy. I mean, maybe around 20% of the stuff that comes in from Fabrication just can’t be assembled. The fabrication is all wrong. As a result, we’ve had to scrap and rework a lot of the stuff. Naturally, this has just shot our quantity variances.

Interpret the variance reports in light of the comments by the Assembly Depart-ment supervisor.

variance interpretationMaster Audio Inc. is a small manufacturer of electronic musical instruments. The plant manager received the following variable factory overhead report for the period:

actual

Budgeted variablefactory overhead at

actual productionControllable

varianceSupplies $28,000 $26,520 $1,480 UPower and light 35,000 33,990 1,010 UIndirect factory wages 26,112 20,400 5,712 UTotal $89,112 $80,910 $8,202 U

Actual units produced: 10,200 (85% of practical capacity)

The plant manager is not pleased with the $8,202 unfavorable variable factory over-head controllable variance and has come to discuss the matter with the controller. The following discussion occurred:

Plant Manager: I just received this factory report for the latest month of operation. I’m not very pleased with these figures. Before these numbers go to headquarters, you and I will need to reach an understanding.

Controller: Go ahead, what’s the problem?

Plant Manager: What’s the problem? Well, everything. Look at the variance. It’s too large. If I understand the accounting approach being used here, you are assuming that my

Chapter 22 Performance Evaluation Using Variances from Standard Costs 1069

Cp 22-4

Page 65: Cases & Projects

costs are variable to the units produced. Thus, as the production volume declines, so should these costs. Well, I don’t believe that these costs are variable at all. I think they are fixed costs. As a result, when we operate below capacity, the costs really don’t go down at all. I’m being penalized for costs I have no control over at all. I need this report to be redone to reflect this fact. If anything, the difference between actual and budget is essentially a volume variance. Listen, I know that you’re a team player. You really need to reconsider your assumptions on this one.

If you were in the controller’s position, how would you respond to the plant manager?

nonmanufacturing performance measures—government

group projectMunicipal governments are discovering that you can control only what you measure. As a result, many municipal governments are introducing nonfinancial performance measures to help improve municipal services. In a group, use the Google search engine to perform a search for “municipal government performance measurement.” Google will provide a list of Internet sites that outline various city efforts in using nonfinancial performance measures. As a group, report on the types of measures used by one of the cities from the search.

Internet Project

1070 Chapter 22 Performance Evaluation Using Variances from Standard Costs

Cp 22-5

Page 66: Cases & Projects

Cases & projects

The Semiconductors Division is able to produce the components used by the Naviga-tional Systems Division. Except for the possible transfer of materials between divisions, no changes are expected in sales and expenses.

instructions1. Would the market price of $1,080 per unit be an appropriate transfer price

for Lockhart Industries, Inc.? Explain.

2. If the Navigational Systems Division purchases 300 units from the Semicon-ductors Division, rather than externally, at a negotiated transfer price of $775 per unit, how much would the income from operations of each division and total company income from operations increase?

3. Prepare condensed divisional income statements for Lockhart Industries, Inc., based on the data in part (2).

4. If a transfer price of $850 per unit is negotiated, how much would the income from operations of each division and total company income from operations increase?

5. a. What is the range of possible negotiated transfer prices that would be acceptable for Lockhart Industries, Inc.?

b. Assuming that the managers of the two divisions cannot agree on a transfer price, what price would you suggest as the transfer price?

ethics and professional conduct in businessC.S. Technology Company has two divisions, the Semiconductor Division and the PC Divi-sion. The PC Division may purchase semiconductors from the Semiconductor Division or from outside suppliers. The Semiconductor Division sells semiconductor products both internally and externally. The market price for semiconductors is $100 per 100 semicon-ductors. Frank Hoffman is the controller of the PC Division, and Lisa Henderson is the controller of the Semiconductor Division. The following conversation took place between Frank and Lisa:

Frank: I hear you are having problems selling semiconductors out of your division. Maybe I can help.

Lisa: You’ve got that right. We’re producing and selling at about 90% of our capacity to outsiders. Last year we were selling 100% of capacity. Would it be possible for your division to pick up some of our excess capacity? After all, we are part of the same company.

Frank: What kind of price could you give me?

Lisa: Well, you know as well as I that we are under strict profit responsibility in our divisions, so I would expect to get market price, $100 for 100 semiconductors.

Frank: I’m not so sure we can swing that. I was expecting a price break from a “sister” division.

Lisa: Hey, I can only take this “sister” stuff so far. If I give you a price break, our profits will fall from last year’s levels. I don’t think I could explain that. I’m sorry, but I must remain firm—market price. After all, it’s only fair—that’s what you would have to pay from an external supplier.

Frank: Fair or not, I think we’ll pass. Sorry we couldn’t have helped.

Was Frank behaving ethically by trying to force the Semiconductor Division into a price break? Comment on Lisa’s reactions.

service department chargesThe Customer Service Department of Morrison Industries asked the Publications De-partment to prepare a brochure for its training program. The Publications Department delivered the brochures and charged the Customer Service Department a rate that was 25% higher than could be obtained from an outside printing company. The policy of the company required the Customer Service Department to use the internal publications group for brochures. The Publications Department claimed that it had a drop in demand for its services during the fiscal year, so it had to charge higher prices in order to recover its payroll and fixed costs.

Chapter 23 Performance Evaluation for Decentralized Operations 1111

Cp 23-1

Cp 23-2

amazzaro
Text Box
Chapter 23
Page 67: Cases & Projects

Should the cost of the brochure be transferred to the Customer Service Depart-ment in order to hold the Customer Service Department head accountable for the cost of the brochure? What changes in policy would you recommend?

evaluating divisional performanceThe three divisions of Tasty Goodness Foods are Snack Goods, Cereal, and Frozen Foods. The divisions are structured as investment centers. The following responsibility reports were prepared for the three divisions for the prior year:

snack Goods Cereal Frozen Foods

Revenues $1,800,000 $2,100,000 $1,650,000Operating expenses 1,039,500 872,000 668,000Income from operations before

service department charges

$ 760,500

$1,228,000

$ 982,000Service department charges:

Promotion $ 250,000 $ 500,000 $ 390,000Legal 114,500 203,000 196,000

$ 364,500 $ 703,000 $ 586,000Income from operations $ 396,000 $ 525,000 $ 396,000Invested assets $1,800,000 $1,750,000 $1,500,000

1. Which division is making the best use of invested assets and thus should be given priority for future capital investments?

2. Assuming that the minimum acceptable rate of return on new projects is 15%, would all investments that produce a return in excess of 15% be accepted by the divisions?

3. Can you identify opportunities for improving the company’s financial performance?

evaluating division performance over timeThe Luxury Vehicle Division of Bimmer Motors Inc. has been experiencing revenue and profit growth during the years 2010–2012. The divisional income statements are provided below.

Bimmer motors inc.Divisional income statements, luxury vehicle Division

For the Years ended December 31, 2010–20122010 2011 2012

Sales $1,050,000 $1,500,000 $1,750,000Cost of goods sold 760,000 1,070,000 1,200,000Gross profit $ 290,000 $ 430,000 $ 550,000Operating expenses 122,000 130,000 165,000Income from operations $ 168,000 $ 300,000 $ 385,000

Assume that there are no charges from service departments. The vice president of the division, Ya-Wen Kelton, is proud of his division’s performance over the last three years. The president of Bimmer Motors Inc., Andrea Canace, is discussing the division’s performance with Ya-Wen, as follows:

Ya-Wen: As you can see, we’ve had a successful three years in the Luxury Vehicle Division.

Andrea: I’m not too sure.

Ya-Wen: What do you mean? Look at our results. Our income from operations has more than doubled, while our profit margins are improving.

Andrea: I am looking at your results. However, your income statements fail to include one very important piece of information; namely, the invested assets. You have been investing a great deal of assets into the division. You had $600,000 in invested assets in 2010, $1,250,000 in 2011, and $1,750,000 in 2012.

Ya-Wen: You are right. I’ve needed the assets in order to upgrade our technologies and expand our operations. The additional assets are one reason we have been able to grow and improve our profit margins. I don’t see that this is a problem.

Andrea: The problem is that we must maintain a 20% rate of return on invested assets.

1112 Chapter 23 Performance Evaluation for Decentralized Operations

Cp 23-3

Cp 23-4

Page 68: Cases & Projects

1. Determine the profit margins for the Luxury Vehicle Division for 2010–2012.

2. Compute the investment turnover for the Luxury Vehicle Division for 2010–2012. Round to two decimal places.

3. Compute the rate of return on investment for the Luxury Vehicle Division for 2010–2012.

4. Evaluate the division’s performance over the 2010–2012 time period. Why was Andrea concerned about the performance?

evaluating division performanceTupelo Industries Inc. is a privately held diversified company with five separate divisions organized as investment centers. A condensed income statement for the Specialty Products Division for the past year, assuming no service department charges, is as follows:

tupelo industries inc.—specialty products Divisionincome statement

For the Year ended December 31, 2012

Sales . . . . . . . . . . . . . . . . . . . . . . . . . . . . . . . . . . . . . . . . . . . . . . . $27,000,000Cost of goods sold . . . . . . . . . . . . . . . . . . . . . . . . . . . . . . . . . . 20,244,000Gross profit . . . . . . . . . . . . . . . . . . . . . . . . . . . . . . . . . . . . . . . . . $ 6,756,000Operating expenses . . . . . . . . . . . . . . . . . . . . . . . . . . . . . . . . . 546,000Income from operations . . . . . . . . . . . . . . . . . . . . . . . . . . . . . $ 6,210,000Invested assets . . . . . . . . . . . . . . . . . . . . . . . . . . . . . . . . . . . . . $33,750,000

The manager of the Specialty Products Division was recently presented with the opportunity to add an additional product line, which would require invested assets of $14,400,000. A projected income statement for the new product line is as follows:

new product lineprojected income statement

For the Year ended December 31, 2012

Sales . . . . . . . . . . . . . . . . . . . . . . . . . . . . . . . . . . . . . . . . . . . . . . . $10,800,000Cost of goods sold . . . . . . . . . . . . . . . . . . . . . . . . . . . . . . . . . . 6,253,500Gross profit . . . . . . . . . . . . . . . . . . . . . . . . . . . . . . . . . . . . . . . . . $ 4,546,500Operating expenses . . . . . . . . . . . . . . . . . . . . . . . . . . . . . . . . . 2,818,500Income from operations . . . . . . . . . . . . . . . . . . . . . . . . . . . . . $ 1,728,000

The Specialty Products Division currently has $33,750,000 in invested assets, and Tu-pelo Industries Inc.’s overall rate of return on investment, including all divisions, is 10%. Each division manager is evaluated on the basis of divisional rate of return on investment. A bonus is paid, in $3,000 increments, for each whole percentage point that the division’s rate of return on investment exceeds the company average.

The president is concerned that the manager of the Specialty Products Division re-jected the addition of the new product line, even though all estimates indicated that the product line would be profitable and would increase overall company income. You have been asked to analyze the possible reasons why the Specialty Products Division manager rejected the new product line.

1. Determine the rate of return on investment for the Specialty Products Division for the past year.

2. Determine the Specialty Products Division manager’s bonus for the past year.

3. Determine the estimated rate of return on investment for the new product line. Round whole percents to one decimal place and investment turnover to two decimal places.

4. Why might the manager of the Specialty Products Division decide to reject the new product line? Support your answer by determining the projected rate of return on investment for 2012, assuming that the new product line was launched in the Specialty Products Division, and 2012 actual operating results were similar to those of 2011.

5. Can you suggest an alternative performance measure for motivating division managers to accept new investment opportunities that would increase the overall company income and rate of return on investment?

Chapter 23 Performance Evaluation for Decentralized Operations 1113

Cp 23-5

Page 69: Cases & Projects

economic value added

Group project Divide into groups and go to the home page of Stern Stewart & Co. at http://www.eva.com. Stern Stewart & Co. is a consulting firm that developed the concept of economic value added (EVA), a measure of corporate and divisional performance, similar to residual income.

Go to the “intellectual property” tab on the Stern Stewart home page to learn about EVA. After reading about EVA, prepare a brief report describing EVA and its claimed ad-vantages. Discuss this research and prepare a brief analysis of this approach to corporate and divisional performance measurement.

Internet Project

1114 Chapter 23 Performance Evaluation for Decentralized Operations

Cp 23-6

Page 70: Cases & Projects

additional business is not expected to affect the domestic sales of the halogen light, and the additional units could be produced using existing capacity.

a. Prepare a differential analysis of the proposed sale to International Lighting Inc.

b. Based on the differential analysis in part (a), should the proposal be accepted?

Product pricing and profit analysis with bottleneck operationsDover Chemical Company produces three products: ethylene, butane, and ester. Each of these products has high demand in the market, and Dover Chemical is able to sell as much as it can produce of all three. The reaction operation is a bottleneck in the process and is running at 100% of capacity. Dover wants to improve chemical operation profitability. The variable conversion cost is $8 per process hour. The fixed cost is $550,000. In addition, the cost analyst was able to determine the following information about the three products:

Ethylene Butane Ester

Budgeted units produced 9,000 9,000 9,000Total process hours per unit 3 3 2Reactor hours per unit 0.75 0.5 1.0Unit selling price $165 $132 $128Direct materials cost per unit $117 $88 $85

The reaction operation is part of the total process for each of these three products. Thus, for example, 1.0 of the 3 hours required to process ethylene is associated with the reactor.

Instructions1. Determine the unit contribution margin for each product.

2. Provide an analysis to determine the relative product profitabilities, assuming that the reactor is a bottleneck.

3. Assume that management wishes to improve profitability by increasing prices on selected products. At what price would ethylene and ester need to be offered in order to produce the same relative profitability as butane?

ObJ. 3

1. Ethylene, $24

Cases & Projects

Product pricingDominique Leone is a cost accountant for Black Box Systems Inc. Orlando Stevens, vice president of marketing, has asked Dominique to meet with representatives of Black Box Systems’ major competitor to discuss product cost data. Orlando indicates that the sharing of these data will enable Black Box Systems to determine a fair and equitable price for its products.

Would it be ethical for Dominique to attend the meeting and share the relevant cost data?

Decision on accepting additional businessA manager of Riviera Sporting Goods Company is considering accepting an order from an overseas customer. This customer has requested an order for 20,000 dozen golf balls at a price of $24 per dozen. The variable cost to manufacture a dozen golf balls is $19 per dozen. The full cost is $26 per dozen. Riviera has a normal selling price of $40 per dozen. Riviera’s plant has just enough excess capacity on the second shift to make the overseas order.

What are some considerations in accepting or rejecting this order?

Accept business at a special priceIf you are not familiar with Priceline.com Inc., go to its Web site. Assume that an individual “names a price” of $80 on Priceline.com for a room in Chicago, Illinois, on May 3. As-sume that May 3 is a Saturday, with low expected room demand in Chicago at a Marriott International, Inc., hotel, so there is excess room capacity. The fully allocated cost per room per day is assumed from hotel records as follows:

Internet Project

1156 Chapter 24 Differential Analysis and Product Pricing

PR 24-6B

CP 24-1

CP 24-2

CP 24-3

amazzaro
Text Box
Chapter 24
Page 71: Cases & Projects

Housekeeping labor cost* $ 36Hotel depreciation expense 40Cost of room supplies (soap, paper, etc.) 7Laundry labor and material cost* 10Cost of desk staff 6Utility cost (mostly air conditioning) 4

Total cost per room per day $103

* Both housekeeping and laundry staff include many part-time workers, so that the workload is variable to demand.

Should Marriott accept the customer bid for a night in Chicago on May 3 at a price of $80?

Cost-plus and target costing conceptsThe following conversation took place between Morris Stein, vice president of marketing, and Cal Rentz, controller of Diamond Computer Company:Morris: I am really excited about our new computer coming out. I think it will be a real market success.

Cal: I’m really glad you think so. I know that our success will be determined by our price. If our price is too high, our competitors will be the ones with the market success.

Morris: Don’t worry about it. We’ll just mark our product cost up by 25% and it will all work out. I know we’ll make money at those markups. By the way, what does the estimated product cost look like?

Cal: Well, there’s the rub. The product cost looks as if it’s going to come in at around $1,200. With a 25% markup, that will give us a selling price of $1,500.

Morris: I see your concern. That’s a little high. Our research indicates that computer prices are dropping and that this type of computer should be selling for around $1,250 when we release it to the market.

Cal: I’m not sure what to do.

Morris: Let me see if I can help. How much of the $1,200 is fixed cost?

Cal: About $200.

Morris: There you go. The fixed cost is sunk. We don’t need to consider it in our pricing decision. If we reduce the prod-uct cost by $200, the new price with a 25% markup would be right at $1,250. Boy, I was really worried for a minute there. I knew something wasn’t right.

a. If you were Cal, how would you respond to Morris’s solution to the pricing problem?

b. How might target costing be used to help solve this pricing dilemma?

Pricing decisions and markup on variable costs

Group Project Many businesses are offering their products and services over the Internet. Some of these companies and their Internet addresses are listed below.

Company Name Internet Address (URL) ProductDelta Air Lines http://www.delta.com Airline ticketsAmazon.com http://www.amazon.com BooksDell Inc. http://www.dell.com Personal computers

a. In groups of three, assign each person in your group to one of the Internet sites listed above. For each site, determine the following:

1. A product (or service) description.

2. A product price.

3. A list of costs that are required to produce and sell the product selected in part (1) as listed in the annual report on SEC Form 10-K.

4. Whether the costs identified in part (3) are fixed costs or variable costs.b. Which of the three products do you believe has the largest markup on variable cost?

Internet Project

Chapter 24 Differential Analysis and Product Pricing 1157 Chapter 24 Differential Analysis and Product Pricing 1157

CP 24-4

CP 24-5

Page 72: Cases & Projects

ProposalCash Payback

PeriodAverage Rate

of ReturnAccept for

Further Analysis RejectABCD

4. For the proposals accepted for further analysis in part (3), compute the net present value. Use a rate of 12% and the present value of $1 table appearing in this chapter (Exhibit 1).

5. Compute the present value index for each of the proposals in part (4). Round to two decimal places.

6. Rank the proposals from most attractive to least attractive, based on the present values of net cash flows computed in part (4).

7. Rank the proposals from most attractive to least attractive, based on the present value indexes computed in part (5). Round to two decimal places.

8. Based on the analyses, comment on the relative attractiveness of the proposals ranked in parts (6) and (7).

Cases & Projects

Ethics and professional conduct in business

Kristen Avery was recently hired as a cost analyst by CareNet Medical Supplies Inc. One of Kristen’s first assignments was to perform a net present value analysis for a new warehouse. Kristen performed the analysis and calculated a present value index of 0.75. The plant manager, Ted Gaines, is very intent on purchasing the warehouse because he believes that more storage space is needed. Ted asks Kristen into his office and the following conversation takes place:

Ted: Kristen, you’re new here, aren’t you?

Kristen: Yes, sir.

Ted: Well, Kristen, I’m not at all pleased with the capital investment analysis that you performed on this new warehouse. I need that warehouse for my production. If I don’t get it, where am I going to place our output?

Kristen: Hopefully with the customer, sir.

Ted: Now don’t get smart with me.

Kristen: No, really, I wasn’t trying to be disrespectful. My analysis does not support constructing a new warehouse. The numbers don’t lie; the warehouse does not meet our investment return targets. In fact, it seems to me that purchasing a warehouse does not add much value to the business. We need to be producing product to satisfy customer orders, not to fill a warehouse.

Ted: Listen, you need to understand something. The headquarters people will not allow me to build the warehouse if the numbers don’t add up. You know as well as I that many assumptions go into your net present value analysis. Why don’t you relax some of your assumptions so that the financial savings will offset the cost?

Kristen: I’m willing to discuss my assumptions with you. Maybe I overlooked something.

Ted: Good. Here’s what I want you to do. I see in your analysis that you don’t project greater sales as a result of the warehouse. It seems to me, if we can store more goods, then we will have more to sell. Thus, logically, a larger ware-house translates into more sales. If you incorporate this into your analysis, I think you’ll see that the numbers will work out. Why don’t you work it through and come back with a new analysis. I’m really counting on you on this one. Let’s get off to a good start together and see if we can get this project accepted.

What is your advice to Kristen?

Personal investment analysisA Masters of Accountancy degree at Central University costs $10,000 for an additional fifth year of education beyond the bachelor’s degree. Assume that all tuition is paid at the beginning of the year. A student considering this investment must evaluate the

1192 Chapter 25 Capital Investment Analysis

CP 25-1

CP 25-2

amazzaro
Text Box
Chapter 25
Page 73: Cases & Projects

present value of cash flows from possessing a graduate degree versus holding only the undergraduate degree. Assume that the average student with an undergraduate degree is expected to earn an annual salary of $48,000 per year (assumed to be paid at the end of the year) for 10 years. Assume that the average student with a graduate Masters of Accountancy degree is expected to earn an annual salary of $63,000 per year (assumed to be paid at the end of the year) for nine years after graduation. Assume a minimum rate of return of 10%.

1. Determine the net present value of cash flows from an undergraduate degree. Use the present value table provided in this chapter in Exhibit 2.

2. Determine the net present value of cash flows from a Masters of Accountancy degree, assuming no salary is earned during the graduate year of schooling.

3. What is the net advantage or disadvantage of pursuing a graduate degree under these assumptions?

Changing pricesInternational Devices Inc. invested $1,000,000 to build a plant in a foreign country. The labor and materials used in production are purchased locally. The plant expansion was estimated to produce an internal rate of return of 20% in U.S. dollar terms. Due to a cur-rency crisis, the currency exchange rate between the local currency and the U.S. dollar doubled from two local units per U.S. dollar to four local units per U.S. dollar.

a. Assume that the plant produced and sold product in the local economy. Explain what impact this change in the currency exchange rate would have on the project’s internal rate of return.

b. Assume that the plant produced product in the local economy but exported the product back to the United States for sale. Explain what impact the change in the currency ex-change rate would have on the project’s internal rate of return under this assumption.

Qualitative issues in investment analysisThe following are some selected quotes from senior executives:

CEO, Worthington Industries (a high technology steel company): “We try to find the best technology, stay ahead of the competition, and serve the customer…. We’ll make any in-vestment that will pay back quickly … but if it is something that we really see as a must down the road, payback is not going to be that important.”

Chairman of Amgen Inc. (a biotech company): “You cannot really run the numbers, do net present value calculations, because the uncertainties are really gigantic … You decide on a project you want to run, and then you run the numbers [as a reality check on your assumptions]. Success in a business like this is much more dependent on tracking rather than on predicting, much more dependent on seeing results over time, tracking and adjusting and readjusting, much more dynamic, much more flexible.”

Chief Financial Officer of Merck & Co., Inc. (a pharmaceutical company): “ … at the individual product level—the development of a successful new product requires on the or-der of $230 million in R&D, spread over more than a decade—discounted cash flow style analysis does not become a factor until development is near the point of manufacturing scale-up effort. Prior to that point, given the uncertainties associated with new product development, it would be lunacy in our business to decide that we know exactly what’s going to happen to a product once it gets out.”

Explain the role of capital investment analysis for these companies.

Net present value methodMetro-Goldwyn-Mayer Studios Inc. (MGM) is a major producer and distributor of theatrical and television filmed entertainment. Regarding theatrical films, MGM states, “Our feature films are exploited through a series of sequential domestic and international distribu-tion channels, typically beginning with theatrical exhibition. Thereafter, feature films are first made available for home video generally six months after theatrical release; for pay television, one year after theatrical release; and for syndication, approximately three to five years after theatrical release.”

Chapter 25 Capital Investment Analysis 1193

CP 25-3

CP 25-4

CP 25-5

Page 74: Cases & Projects

Assume that MGM produces a film during early 2012 at a cost of $300 million, and releases it halfway through the year. During the last half of 2012, the film earns revenues of $350 million at the box office. The film requires $75 million of advertising during the release. One year later, by the end of 2013, the film is expected to earn MGM net cash flows from home video sales of $45 million. By the end of 2014, the film is expected to earn MGM $30 million from pay TV; and by the end of 2015, the film is expected to earn $10 million from syndication.a. Determine the net present value of the film as of the beginning of 2012 if the desired

rate of return is 20%. To simplify present value calculations, assume all annual net cash flows occur at the end of each year. Use the table of the present value of $1 appearing in Exhibit 1 of this chapter. Round to the nearest whole million dollars.

b. Under the assumptions provided here, is the film expected to be financially successful?

Capital investment analysis

group ProjectIn one group, find a local business, such as a copy shop, that rents time on desktop computers for an hourly rate. Determine the hourly rate. In the other group, determine the price of a mid-range desktop computer at http://www.dell.com. Combine this infor-mation from the two groups and perform a capital budgeting analysis. Assume that one student will use the computer for 40 hours per semester for the next three years. Also assume that the minimum rate of return is 10%. Use the interest tables in Appendix A in performing your analysis. [Hint: Use the appropriate present value of an annuity of $1 factor for 5% compounded for six semiannual periods (periods = 6).]

Does your analysis support the student purchasing the computer?

Internet Project

1194 Chapter 25 Capital Investment Analysis

CP 25-6

Page 75: Cases & Projects

Product costing and decision analysis for a passenger airlineBlue Skies Airline provides passenger airline service, using small jets. The airline connects four major cities: Atlanta, Cincinnati, Chicago, and Los Angeles. The company expects to fly 140,000 miles during a month. The following costs are budgeted for a month:

Fuel $1,450,000Ground personnel 693,250Crew salaries 710,000Depreciation 360,000 _________ Total costs $3,213,250 _________ _________

Blue Skies management wishes to assign these costs to individual flights in order to gauge the profitability of its service offerings. The following activity bases were identified with the budgeted costs:

Airline Cost Activity Base

Fuel, crew, and depreciation costs Number of miles flownGround personnel Number of arrivals and departures at an airport

The size of the company’s ground operation in each city is determined by the size of the workforce. The following monthly data are available from corporate records for each terminal operation:

Ground Personnel Number of Terminal City Cost Arrivals/Departures

Atlanta $224,750 290Cincinnati 84,000 120Chicago 114,800 140Los Angeles 269,700 310 ________ ___ Total $693,250 860 ________ ___ ________ ___

Three recent representative flights have been selected for the profitability study. Their characteristics are as follows:

Number of Ticket Price Description Miles Flown Passengers per Passenger

Flight 101 Atlanta to LA 1,850 70 $650Flight 102 Chicago to Atlanta 600 45 400Flight 103 Atlanta to Cincinnati 350 20 350

Instructions1. Determine the fuel, crew, and depreciation cost per mile flown.2. Determine the cost per arrival or departure by terminal city.3. Use the information in (1) and (2) to construct a profitability report for the three flights.

Each flight has a single arrival and departure to its origin and destination city pairs.4. Evaluate flight profitability by determining the break-even number of passengers re-

quired for each flight assuming all the costs of a flight are fixed. Round to the nearest whole number.

ObJ. 6

✔ 3. Flight 102 income from operations, $5,605

Cases & Projects

Ethics and professional conduct in businessThe controller of XTek Systems Inc. devised a new costing system based on tracing the cost of activities to products. The controller was able to measure post-manufacturing activities, such as selling, promotional, and distribution activities, and allocate these activities to products in order to have a more complete view of the company’s product costs. This effort

1238 Chapter 26 Cost Allocation and Activity-Based Costing

PR 26-6b

CP 26-1

amazzaro
Text Box
Chapter 26
Page 76: Cases & Projects

produced better strategic information about the relative profitability of product lines. In addition, the controller used the same product cost information for inventory valuation on the financial statements. Surprisingly, the controller discovered that the company’s reported net income was larger under this scheme than under the traditional costing approach.

Why was the net income larger, and how would you react to the controller’s action?

Identifying product cost distortionBreeze Beverage Company manufactures soft drinks. Information about two products is as follows:

Sales Price Gross Profit Volume per Case per Case

Dr. Kola 800,000 cases $30 $12Tahiti Punch 10,000 cases 30 12

It is known that both products have the same direct materials and direct labor costs per case. Breeze Beverage allocates factory overhead to products by using a single plantwide factory overhead rate, based on direct labor cost. Additional information about the two products is as follows:

Dr. Kola: Requires minor process preparation and sterilization prior to processing. The ingredients are acquired locally. The formulation is simple, and it is easy to maintain quality. Lastly, the product is sold in large bulk (full truckload) orders.

Tahiti Punch: Requires extensive process preparation and sterilization prior to processing. The ingredients are from Jamaica, requiring complex import controls. The formulation is complex, and it is thus difficult to maintain quality. Lastly, the product is sold in small (less than full truckload) orders.

Explain the product profitability report in light of the additional data.

Activity-based costingWells Fargo Insurance Services (WFIS) is an insurance brokerage company that clas sified insurance products as either “easy” or “difficult.” Easy and difficult products were defined as follows:

Easy: Electronic claims, few inquiries, mature product

Difficult: Paper claims, complex claims to process, many inquiries, a new product with complex options

The company originally allocated processing and service expenses on the basis of revenue. Under this traditional allocation approach, the product profitability report re-vealed the following:

Easy Difficult Product Product Total

Revenue $600 $400 $1,000Processing and service expenses 420 280 700 ____ ____ _____Income from operations $180 $120 $ 300 ____ ____ _____ ____ ____ _____Operating income margin 30% 30% 30%

WFIS decided to use activity-based costing to allocate the processing and service ex-penses. The following activity-based costing analysis of the same data illustrates a much different profit picture for the two types of products.

Easy Difficult Product Product Total

Revenue $600 $ 400 $1,000Processing and service expenses 183 517 700 ____ ____ _____Income from operations $417 $(117) $ 300 ____ ____ _____ ____ ____ _____Operating income margin 70% (29%) 30%

Explain why the activity-based profitability report reveals different information from the traditional sales allocation report.

Source: Dan Patras and Kevin Clancy, “ABC in the Service Industry: Product Line Profitability at Acordia, Inc.” As Easy as ABC Newsletter, Issue 12, Spring 1993.

Chapter 26 Cost Allocation and Activity-Based Costing 1239

CP 26-2

CP 26-3

Page 77: Cases & Projects

Using a product profitability report to guide strategic decisionsThe controller of Fidelity Sounds Inc. prepared the following product profitability report for management, using activity-based costing methods for allocating both the factory overhead and the marketing expenses. As such, the controller has confidence in the accuracy of this report. In addition, the controller interviewed the vice president of marketing, who indicated that the floor loudspeakers were an older product that was highly recognized in the marketplace. The ribbon loudspeakers were a new product that was recently launched. The ribbon loudspeakers are a new technology that have no competition in the marketplace, and it is hoped that they will become an important future addition to the company’s product portfolio. Initial indications are that the product is well received by customers. The controller believes that the manufactur-ing costs for all three products are in line with expectations.

Floor Bookshelf Ribbon Loudspeakers Loudspeakers Loudspeakers Totals

Sales $1,200,000 $800,000 $600,000 $2,600,000Less cost of goods sold 816,000 448,000 552,000 1,816,000 ________ _______ _______ _________Gross profit $ 384,000 $352,000 $ 48,000 $ 784,000Less marketing expenses 480,000 72,000 42,000 594,000 ________ _______ _______ _________Income from operations $ (96,000) $280,000 $ 6,000 $ 190,000 ________ _______ _______ _________ ________ _______ _______ _________

1. Calculate the gross profit and income from operations to sales ratios for each product.2. Write a memo using the product profitability report and the calculations in (1) to make

recommendations to management with respect to strategies for the three products.

Product cost distortionAshton Mills, president of Smart Calc Inc., was reviewing the product profitability reports with the controller, Andrew Scott. The following conversation took place:

Ashton: I’ve been reviewing the product profitability reports. Our high-volume calculator, the T-100, appears to be unprofitable, while some of our lower-volume specialty calculators in the T-900 series appear to be very profitable. These results do not make sense to me. How are the product profits determined?

Andrew: First, we identify the revenues associated with each product line. This information comes directly from our sales order system and is very accurate. Next, we identify the direct materials and direct labor associated with making each of the calculators. Again, this information is very accurate. The final cost that must be considered is the factory overhead. Factory overhead is allocated to the products, based on the direct labor hours used to assemble the calculator.

Ashton: What about distribution, promotion, and other post-manufacturing costs that can be associated with the product?

Andrew: According to generally accepted accounting principles, we expense them in the period that they are incurred and do not treat them as product costs.

Ashton: Another thing, you say that you allocate factory overhead according to direct labor hours. Yet I know that the T-900 series specialty products have very low volumes but require extensive engineering, testing, and materials management effort. They are our newer, more complex products. It seems that these sources of factory overhead will end up being allocated to the T-100 line because it is the high- volume and therefore high direct labor hour product. Yet the T-100 line is easy to make and requires very little support from our engineering, testing, and materi-als management personnel.

Andrew: I’m not too sure. I do know that our product costing approach is similar to that used by many different types of companies. I don’t think we could all be wrong.

Is Ashton Mills’ concern valid, and how might Andrew Scott redesign the cost allocation system to address Ashton’s concern?

Allocating bank administrative costsBanks have a variety of products, such as savings accounts, checking accounts, certificates of deposit (CDs), and loans. Assume that you were assigned the task of determining the administrative costs of “savings accounts” as a complete product line. What are some of the activities associated with savings accounts? In answering this question, consider the activities that you might perform with your savings account. For each activity, what would be an activity base that could be used to allocate the activity cost to the savings account product line?

1240 Chapter 26 Cost Allocation and Activity-Based Costing

CP 26-4

CP 26-5

CP 26-6

Page 78: Cases & Projects

Cases & Projects

3. Journalize the summary transactions (a) through (d).

4. Determine the ending balance in Raw and In Process Inventory and Finished Goods Inventory.

5. How does the accounting in a JIT environment differ from traditional accounting?

Pareto chart and cost of quality report-manufacturing companyThe president of Cardio Care Exercise Equipment Co. has been concerned about the growth in costs over the last several years. The president asked the controller to perform an activity analysis to gain a better insight into these costs. The activity analysis revealed the following:

Activities Activity CostCorrecting invoice errors $ 7,200Disposing of incoming materials with poor quality 21,600Disposing of scrap 43,200Expediting late production 36,000Final inspection 28,800Inspecting incoming materials 7,200Inspecting work in process 32,400Preventive machine maintenance 21,600Producing product 140,400Responding to customer quality complaints 21,600 Total $360,000

The production process is complicated by quality problems, requiring the production manager to expedite production and dispose of scrap.

Instructions1. Prepare a Pareto chart of the company activities.

2. Use the activity cost information to determine the percentages of total costs that are prevention, appraisal, internal failure, external failure, and not costs of quality (pro-ducing product).

3. Determine the percentages of total costs that are value- and non-value-added.

4. Interpret the information.

ObJ. 4

3. Non-value-added, 36%

Ethics and professional conduct in businessIn August, Selene Company introduced a new performance measurement system in manu-facturing operations. One of the new performance measures was lead time. The lead time was determined by tagging a random sample of items with a log sheet throughout the month. This log sheet recorded the time that the item started and the time that it ended production, as well as all steps in between. The controller collected the log sheets and calculated the average lead time of the tagged products. This number was reported to central management and was used to evaluate the performance of the plant manager. The plant was under extreme pressure to reduce lead time because of poor lead time results reported in June.

Chapter 27 Cost Management for Just-in-Time Environments 1277

PR 27-4B

CP 27-1

amazzaro
Text Box
Chapter 27
Page 79: Cases & Projects

The following memo was intercepted by the controller.

Date: September 1To: Hourly EmployeesFrom: Plant Manager

During last month, you noticed that some of the products were tagged with a log sheet. This sheet records the time that a product enters production and the time that it leaves production. The difference between these two times is termed the “lead time.” Our plant is evaluated on improving lead time. From now on, I ask all of you to keep an eye out for the tagged items. When you receive a tagged item, it is to receive special attention. Work on that item first, and then immediately move it to the next operation. Under no circumstances should tagged items wait on any other work that you have. Naturally, report accurate information. I insist that you record the correct times on the log sheet as the product goes through your operations.

How should the controller respond to this discovery?

Just-in-time principlesComfort Space Inc. manufactures electric space heaters. While the CEO, Toma Zulim, is visiting the production facility, the following conversation takes place with the plant manager, Raj Chandar:

Toma: As I walk around the facility, I can’t help noticing all the materials inventories. What’s going on?

Raj: I have found our suppliers to be very unreliable in meeting their delivery commitments. Thus, I keep a lot of materials on hand so as to not risk running out and shutting down production.

Toma: Not only do I see a lot of materials inventory, but there also seems to be a lot of finished goods inventory on hand. Why is this?

Raj: As you know, I am evaluated on maintaining a low cost per unit. The one way that I am able to reduce my unit costs is by producing as many space heaters as possible. This allows me to spread my fixed costs over a larger base. When orders are down, the excess production builds up as inventory, as we are seeing now. But don’t worry—I’m really keeping our unit costs down this way.

Toma: I’m not so sure. It seems that this inventory must cost us something.

Raj: Not really. I’ll eventually use the materials and we’ll eventually sell the finished goods. By keeping the plant busy, I’m using our plant assets wisely. This is reflected in the low unit costs that I’m able to maintain.

If you were Toma Zulim, how would you respond to Raj Chandar? What recom-mendations would you provide Raj Chandar?

Just-in-time principlesHyperion Concepts Inc. prepared the following performance graphs for the prior year:

2 3 4 5 6 7 8 9 10 11

Day

s

Months

Total Manufacturing Lead Time

12

0

10

20

30

40

50

60

70

1

1278 Chapter 27 Cost Management for Just-in-Time Environments

CP 27-2

CP 27-3

Page 80: Cases & Projects

2 3 4 5 6 7 8 9 10 11Months

Total Inventory Dollars (in 000s)

121

0

20

40

60

80

100

120

140

$160

2 3 4 5 6 7 8 9 10 11

Perc

ent

Months

Percent of Sales Orders Filled on Time

12

0

10

20

30

40

50

60

70

90

80

100%

What do these appear to indicate?

Value-added and non-value-added activity costsTurner Company prepared the following factory overhead report from its general ledger:

Indirect labor $200,000Fringe benefits 30,000Supplies 70,000Depreciation 50,000 Total $350,000

The management of Turner Company was dissatisfied with this report and asked the controller to prepare an activity analysis of the same information. This activity analysis was as follows:

Processing sales orders $ 66,500 19%Disposing of scrap 77,000 22Expediting work orders 70,000 20Producing parts 42,000 12Resolving supplier quality problems 45,500 13Reissuing corrected purchase orders 35,000 10Expediting customer orders 14,000 4 Total $350,000 100%

Interpret the activity analysis by identifying value-added and non-value-added activity costs. How does the activity cost report differ from the general ledger report?

Lead time

Group ProjectIn groups of two to four people, visit a sit-down restaurant and do a lead time study. If more than one group chooses to visit the same restaurant, choose different times for your visits. Note the time when you walk in the door of the restaurant and the time when you walk out the door after you have eaten. The difference between these two times is the total lead time of your restaurant experience. While in the restaurant, determine the time spent on non-value-added time, such as wait time, and the time spent on value-added eating time. Note the various activities and the time required to perform each activity during your visit to the restaurant. Compare your analyses, identifying possible reasons for differences in the times recorded by groups that visited the same restaurant.

Chapter 27 Cost Management for Just-in-Time Environments 1279

CP 27-4

CP 27-5